Respiratory Disorders - ML8

Ace your homework & exams now with Quizwiz!

A client with a nasotracheal tube needs to be suctioned. What is the length of time the nurse should apply the suction for each pass of the catheter? 40 to 45 seconds 1 to 5 seconds 20 to 25 seconds 10 to 15 seconds

10 to 15 seconds Suction should be applied for 10 to 15 seconds for each pass of the catheter. Suctioning for longer than 15 seconds removes oxygen from the respiratory tract and cause hypoxemia. Suctioning less than 10 seconds would not be adequate to remove the secretions.

A client with bronchitis is ordered 300 mg of liquid guaifenesin every 4 hours. The container indicates that there is 200 mg/5 mL. How many milliliters should the nurse administer per dose? Record your answer using one decimal place.

7.5 The following formula is used to calculate the drug dosage: Dose on hand/Quantity on hand = Dose desired/X. Plug in the values for this equation: 200 mg/5 mL = 300 mg/X = 7.5 mL

A nurse is teaching a client about using an incentive spirometer. Which statement by the nurse is correct? "Before you do the exercise, I'll give you pain medication if you need it." "Don't use the incentive spirometer more than 5 times every hour." "Breathe in and out quickly." "You need to start using the incentive spirometer 2 days after surgery."

"Before you do the exercise, I'll give you pain medication if you need it." The nurse should assess the client's pain level before the client does incentive spirometry exercises and administer pain medication as needed. Doing so helps the client take deeper breaths and help prevents atelectasis. The client should breathe in slowly and steadily, and hold their breath for 3 seconds after inhalation. The client should start doing incentive spirometry immediately after surgery and aim to do 10 incentive spirometry breaths every hour.

A nurse is providing discharge teaching for a client who had a laryngectomy. Which instruction should the nurse include in the teaching? "Cover the stoma with a loose plastic cloth whenever you shower or bathe." "Keep the humidity in your house low." "Cover the stoma with your hand to prevent anything from entering it." "Swimming is good exercise as long as you don't go under water."

"Cover the stoma with a loose plastic cloth whenever you shower or bathe." The nurse should instruct the client to gently cover the stoma with a loose plastic bib when showering or bathing to prevent water from entering the stoma. The client should cover the stoma with a loose-fitting, not tight, cloth to protect it. Covering the stoma with the hand is not practical and short term at best. The client should keep the house humidified to prevent irritation of the stoma that can occur in low humidity. The client should avoid swimming at all; the activity is too risky, because it's possible for water to enter the stoma and then enter the client's lung, causing drowning without submerging the face.

The nurse is educating a client about allergy management at home. What client statements indicate no further teaching is required? Select all that apply. "I have pull shades on all of my windows." "I bought a wooden chair for my living room." "I picked out a new tufted bedspread for my bed." "I only let my dog sleep with me every other day." "I will vacuum my floors once a week."

"I have pull shades on all of my windows." "I bought a wooden chair for my living room." Using pull shades on windows and using steam for heating will help reduce environmental allergens such as dust. Rugs on floors will hold allergens in and floors need to be vacuumed every day. Dogs may bring allergens into the client's home, especially when in close proximity while sleeping.

In preparation for discharge, the nurse teaches the mother of an infant diagnosed with bronchiolitis about the condition and its treatment. Which statement by the mother indicates successful teaching? "I need to be sure to take my child's temperature every day." "Next time my child gets a cold I need to listen to the chest." "I need to wash my hands more often." "I hope I do not get a cold from my child."

"I need to wash my hands more often." Handwashing is the best way to prevent respiratory illnesses and the spread of disease. Bronchiolitis, a viral infection primarily affecting the bronchioles, causes swelling and mucus accumulation of the lumina and subsequent hyperinflation of the lung with air trapping. It is transmitted primarily by direct contact with respiratory secretions as a result of eye-to-hand or nose-to-hand contact or from contaminated fomites. Therefore, handwashing minimizes the risk for transmission. Taking the child's temperature is not appropriate in most cases. As long as the child is getting better, taking the temperature will not be helpful. The mother's statement that she hopes she does not get a cold from her child does not indicate understanding of what to do after discharge. For most parents, listening to the child's chest would not be helpful because the parents would not know what they were listening for. Rather, watching for an increased respiratory rate, fever, or evidence of poor eating or drinking would be more helpful in alerting the parent to potential illness.

A nurse is caring for a client who has a history of sleep apnea. The client understands the disease process when they say "I should become involved in a weight loss program." "I should eat a high-protein diet." "I need to keep my inhaler at the bedside." "I should sleep on my side all night long."

"I should become involved in a weight loss program." Obesity and decreased pharyngeal muscle tone commonly contribute to sleep apnea; the client may need to become involved in a weight loss program. Using an inhaler won't alleviate sleep apnea, and the physician probably wouldn't order an inhaler unless the client had other respiratory complications. A high-protein diet and sleeping on the side aren't treatment factors associated with sleep apnea.

A client admitted to the facility for treatment for tuberculosis receives instructions about the disease. Which statement made by the client indicates the need for further instruction? "I'll stay in isolation for 6 weeks." "This disease may come back later if I am under stress." "I'll always have a positive test for tuberculosis." "I'll have to take the medication for up to a year."

"I'll stay in isolation for 6 weeks." The client requires additional teaching if they state that they'll be in isolation for 6 weeks. The client needs to be in isolation for 2 weeks, not 6, while taking the tuberculosis drugs. After 2 weeks of antitubercular therapy, the client is no longer considered contagious. The client needs to receive the drugs for 9 months to a year. The client will be positive when tested and if they are sick or under some stress they could have a relapse of the disease.

A client who has started therapy for drug-resistant tuberculosis demonstrates understanding of tuberculosis transmission by saying: "I'm clear when my chest X-ray is negative after 1 month of medication." "I'll stop being contagious when I have a negative acid-fast bacilli test." "I'm contagious as long as I have night sweats." "My tuberculosis isn't contagious after I take the medication for 24 hours."

"I'll stop being contagious when I have a negative acid-fast bacilli test." A client with drug-resistant tuberculosis is not contagious when the client has had a negative acid-fast test. A client with nonresistant tuberculosis is no longer considered contagious when there is clinical evidence of decreased infection, such as significantly decreased coughing and fewer organisms on sputum smears. The medication may not produce a negative acid-fast test result for several days. The client will not have a clear chest X-ray for several months after starting treatment. Night sweats are a sign of tuberculosis, but they do not indicate whether the client is contagious.

The nursing student is explaining sputum for cytology procedure to a client. Which statements require the nurse to intervene? Select all that apply. "You will need to cough and deep breathe to try and mobilize your sputum." "Just cough into the tissue, then transfer the specimen to this container." "If you just put saliva in the container it will be fine." "If you cannot expectorate sputum, we can suction it easily." "You can try to expectorate the sputum after dinner tonight."

"If you cannot expectorate sputum, we can suction it easily." "You can try to expectorate the sputum after dinner tonight." "If you just put saliva in the container it will be fine." "Just cough into the tissue, then transfer the specimen to this container." Instruct the client to rinse the mouth with water to reduce specimen contamination, take three slow, deep breaths and then to cough deeply from a maximal inspiration. Have the client repeat the procedure as necessary until the client has produced sputum. When the client has mobilized the sputum, instruct the client to expectorate directly into a sterile specimen container without touching the inside or rim of the container. Have the client continue producing and expectorating sputum until the amount totals at least 5 mL, if possible. Assess the sputum specimen to ensure that it's actually sputum and not saliva because saliva produces inaccurate test results. Sputum appears thick and opaque, while saliva appears thin, clear, and watery. When a client can't produce an adequate sputum specimen, additional treatments may need to be performed to help mobilize secretions. These treatments include nebulizer use, hydration, deep-breathing exercises, chest percussion, and postural drainage.

A nurse recognizes that a client with tuberculosis needs further teaching when the client states: "It will be necessary for the people I work with to take medication." "The people I have contact with at work should be checked also." "I'll have to take these medications for 9 to 12 months." "I'll need to have scheduled laboratory tests while I'm on the medication."

"It will be necessary for the people I work with to take medication." The client requires additional teaching if he states that coworkers will need to take medication. If exposed and testing positive, medications would be required for coworkers. Contacts need to be tested for tuberculosis. However, a person in close contact with a person who's infectious is at greatest risk and should be definitely be checked. The client demonstrates effective teaching when stating a need to take medications for 9 to 12 months and that required laboratory tests while on medication.

The nurse is reconciling the prescriptions for a client diagnosed recently with pulmonary tuberculosis who is being admitted to the hospital for a total hip replacement (see medication prescription sheet). The client asks if it is necessary to take all of these medications while in the hospital. What should the nurse tell the client? "It's important to continue to take the medications because the combination of drugs prevents bacterial resistance." "I'll ask your health care provider (HCP) to review the prescriptions for a duplication between isoniazid and ethambutol." "I'll ask the pharmacist to check for drug interactions between the rifampin and isoniazid." "I can't discontinue any of these drugs until you can eat solid foods."

"It's important to continue to take the medications because the combination of drugs prevents bacterial resistance." The nurse should tell the client that it is necessary to take all of these medications because combination drug therapy prevents bacterial resistance; they will be administered throughout the hospitalization to maintain blood levels. The HCP will review the prescriptions per hospital policy because the client is being admitted to the hospital; there is no duplication between any of the drugs being prescribed for this client. It is not necessary to ask the pharmacist to check for drug interactions as these drugs are commonly used together.

Which statement by the mother of a neonate diagnosed with bronchopulmonary dysplasia (BPD) indicates effective teaching? "Bronchodilators can cure my baby's condition." "BPD is an acute disease that can be treated with antibiotics." "My baby may require long-term respiratory support." "My baby may have seizures later on in life because of this condition."

"My baby may require long-term respiratory support." BPD is a chronic illness that may require prolonged hospitalization and permanent assisted ventilation. The disease typically occurs in compromised very-low-birth-weight neonates who require oxygen therapy and assisted ventilation for treatment of respiratory distress syndrome. The cause is multifactorial, and the disease has four stages. The neonate's activities may be limited by the disease. Antibiotics may be prescribed and bronchodilators may be used, but these medications will not cure the chronic disease state. Seizure activity is associated with periventricular-intraventricular hemorrhage, not BPD.

The nurse is instructing a client with chronic obstructive pulmonary disease how to do pursed-lip breathing. In which order from first to last should the nurse explain the steps to the client? All options must be used.

"Relax your neck and shoulder muscles." "Breathe in normally through your nose for two counts (while counting to yourself, one, two)." "Pucker your lips as if you were going to whistle." "Breathe out slowly through pursed lips for four counts (while counting to yourself, one, two, three, four)." The nurse should first instruct the client to relax the neck and the shoulders and then take several normal breaths. After taking a breath in, the client should pucker the lips and finally breathe out through pursed lips.

A client who comes to the clinic reporting low-grade afternoon fevers, night sweats, and a productive cough is diagnosed with tuberculosis. Which statement by the nurse would be most appropriate during an initial assessment? "I'll call the healthcare provider to answer any questions you have." "You shouldn't be embarrassed that you have tuberculosis." "Tell me how you feel about the diagnosis of tuberculosis." "The treatment plan for tuberculosis is quite challenging."

"Tell me how you feel about the diagnosis of tuberculosis." Asking the client about how they feel encourages the client to express feelings about the diagnosis and sets the foundation for a therapeutic relationship. Deferring all questions to the healthcare provider will not build a trusting relationship with the client. Telling the client the treatment plan is challenging, while true, may induce unnecessary fear or destroy hope. Telling the client to not be embarrassed may make the client uncomfortable and may hinder the therapeutic relationship.

A client has been diagnosed with legionellosis (Legionnaires' disease). The client asks, "How did I get this?" Which response by the nurse is the most accurate? "You may have swallowed bacteria-contaminated water." "You inhaled the bacteria in a smoke-filled room. " "The bacteria are inhaled from contaminated water droplets." "As ceiling fans circulate, bacteria are dispersed into the air."

"The bacteria are inhaled from contaminated water droplets." Legionellosis is a pneumonia caused by the bacterium Legionella pneumophilia that thrives in water that is 95° to 115° F (35° to 46° C). When a building's hot water plumbing has water at this temperature, the bacteria thrive; then they may be transmitted via inhalation from air conditioning, showers, spas, and whirlpools. The bacteria are not transmitted via smoke or ceiling fan blades or by swallowing contaminated water.

An older adult's daughter is asking about the follow-up evaluation for her father after his pneumonectomy for primary lung cancer. What should the nurse tell the daughter? "The follow-up for your father will be a chest X-ray every 6 months." "The follow-up for your father will be a chest X-ray and a computed tomography scan of the abdomen every year." "No follow-up is needed at this time." "The usual follow-up is chest X-ray and liver function tests every 3 months."

"The follow-up for your father will be a chest X-ray every 6 months." Follow-up generally involves semiannual chest radiographs. Recurrence usually occurs locally in the lungs and may be identified on chest radiographs. Follow-up after cancer treatment is an important component of the treatment plan. Serum markers (liver function tests) have not been shown to detect recurrence of lung cancer. There are no data to support the need for an abdominal computed tomography scan.

When administering atropine sulfate preoperatively to a client scheduled for lung surgery, what should the nurse tell the client? "This medicine will help you relax." "This medicine will make you drowsy." "This medicine will make your mouth feel dry." "This medicine will reduce the risk of postoperative infection."

"This medicine will make your mouth feel dry." Atropine is an anticholinergic drug that decreases mucus secretions in the respiratory tract and dries the mucus membranes of the mouth, nose, pharynx, and bronchi. Atropine does not cause drowsiness or relaxation. Moderate to large doses cause tachycardia and palpitations. Large doses cause excitement and manic behavior. Atropine does not reduce the risk of postoperative infection.

A client with chronic obstructive pulmonary disease (COPD) and cor pulmonale is being prepared for discharge. The nurse should provide which instruction? "Limit yourself to smoking only 2 cigarettes per day." "Weigh yourself daily and report a gain of 2 lb (0.91 kg) in 1 day." "Maintain bed rest." "Eat a high-sodium diet."

"Weigh yourself daily and report a gain of 2 lb (0.91 kg) in 1 day." The nurse should instruct the client to weigh themselves daily and report a gain of 2 lb (0.91 kg) in 1 day. COPD causes pulmonary hypertension, leading to right-sided heart failure or cor pulmonale. The resultant venous congestion causes dependent edema. A weight gain may further stress the respiratory system and worsen the client's condition. The nurse should also instruct the client to eat a low-sodium diet to avoid fluid retention and engage in moderate exercise to avoid muscle atrophy. The client shouldn't smoke at all.

An older adult has asthma and asks the nurse about taking the pneumonia vaccine. The nurse should tell the client: "You don't need the vaccine unless you're exposed to pneumonia." "You should receive the vaccine." "You shouldn't have the vaccine because it's contraindicated in asthma." "You'll need the vaccine only if you have frequent asthma attacks."

"You should receive the vaccine." Elderly clients, especially those with existing respiratory compromise, are good candidates for the pneumonia vaccine. This vaccine provides immunity and prophylaxis against pneumococcal pneumonia or bacteremia in adults or children at risk. The nurse should encourage the client to take the vaccine. Taking the vaccine is not related to frequency of asthma attacks, and the vaccine is not contraindicated for clients with asthma.

The nurse is caring for a client with emphysema. The client asks about the reason for persistent respiratory acidosis. What is the best response by the nurse? "Your respiratory acidosis is caused by narrowed bronchioles and decreased oxygen." "You experience this because you have an increased work of breathing." "Your alveoli have lost elasticity, which causes retained carbon dioxide." "You have an increase in production of mucus, which causes a lack of oxygen."

"Your alveoli have lost elasticity, which causes retained carbon dioxide." Emphysema causes a loss of alveolar elasticity. The alveoli become hyperinflated with retained carbon dioxide, which leads to chronic respiratory acidosis. While the client with emphysema will experience increased work of breathing, it is the retained carbon dioxide that causes the respiratory acidosis. Narrowed bronchioles and increased mucus production are characteristic of chronic bronchitis, which causes hypoxemia, not respiratory acidosis.

The health care provider prescribes 0.4 mg of atropine sulfate and 75 mg of meperidine hydrochloride to be given intramuscularly to a client 1 hour before surgery. The stock ampule of atropine contains 0.8 mg/mL, and the stock ampule of meperidine contains 100 mg/mL. The two drugs are compatible and can be drawn up in one syringe. What is the combined volume of medication in the syringe? 1.75 mL 0.75 mL 1.50 mL 1.25 mL

1.25 mL The correct amount to administer is determined by using ratios, as follows:0.8 mg/1 mL = 0.4 mg/x mL0.8/x = 0.4x = 0.5 mL of atropine sulfate100 mg/1 mL = 75 mg/x mL100/x = 75x = 0.75 mL of meperidine hydrochloride0.5 mL of atropine + 0.75 mL of meperidine hydrochloride = 1.25 mL total

A client admitted with pneumonia has a history of lung cancer and heart failure. A nurse caring for this client recognizes that they should maintain adequate fluid intake to keep secretions thin for ease in expectoration. The amount of fluid intake this client should maintain is unspecified. 2 L. 1.4 L. 3 L.

1.4 L. Clients need to keep their secretions thin by drinking 2 to 3 L of clear liquids per day. In clients with heart failure, fluid intake shouldn't exceed 1.5 L daily.

The nurse is suctioning a client who had a laryngectomy. What is the maximum amount of time the nurse should suction the client? 25 seconds 30 seconds 20 seconds 10 seconds

10 seconds A client should be suctioned for no longer than 10 seconds at a time. Suctioning for longer than 10 seconds may reduce the client's oxygen level so much that he becomes hypoxic.

A client with asthma is receiving a theophylline preparation to promote bronchodilation. Because of the risk of drug toxicity, the nurse must monitor the client's serum theophylline level closely. The nurse knows that the therapeutic theophylline concentration falls within which range? 2 to 5 mcg/ml 21 to 25 mcg/ml 10 to 20 mcg/ml 5 to 10 mcg/ml

10 to 20 mcg/ml The therapeutic serum theophylline concentration ranges from 10 to 20 mcg/ml. Values below 10 mcg/ml aren't therapeutic. Concentrations above 20 mcg/ml are considered toxic.

After receiving an oral dose of codeine for an intractable cough, a client asks the nurse, "How long will it take for this drug to work?" How should the nurse respond? 1 hour 4 hours 2.5 hours 30 minutes

30 minutes Codeine's onset of action is 30 minutes. Its peak concentration occurs in about 1 hour; its half-life, in 2.5 hours; and its duration of action is 4 to 6 hours.

A client undergoes surgery to repair lung injuries. Postoperative orders include the transfusion of one unit of packed red blood cells at a rate of 60 ml/hour. How long will this transfusion take to infuse? 6 hours 8 hours 4 hours 2 hours

4 hours One unit of packed red blood cells is about 250 mL. If the blood is delivered at a rate of 60 mL/h, it will take about 4 hours to infuse the entire unit. The transfusion of a single unit of packed red blood cells should not exceed 4 hours to prevent the growth of bacteria and minimize the risk of septicemia.

The nurse monitors a client following the insertion of a chest tube for a hemopneumothorax. Which observation should the nurse report to the healthcare provider? 600 mL of blood in the collection chamber in 1 hour subcutaneous emphysema at the insertion site intermittent bubbling in the water seal chamber continuous bubbling in the suction-control chamber

600 mL of blood in the collection chamber in 1 hour A blood loss of 600 mL may place the client in danger of developing hypovolemic shock. All of the other choices are normally expected with a chest tube.

While reviewing the arterial blood gas values of a client with emphysema, the nurse should identify which PaCO2 values as indicating the need for immediate intervention? 60 mm Hg 45 mm Hg 35 mm Hg 80 mm Hg

80 mm Hg Although normal PaCO2 values range from 35 to 45 mm Hg, the client with long-standing emphysema has chronic carbon dioxide retention, leading to elevated PaCO2 levels. A PaCO2 level of 80 mm Hg is life threatening and always requires immediate intervention, possibly mechanical ventilation, to reduce the PaCO2 level.The client with emphysema and a PaCO2 level of 60 mm Hg may not be in immediate danger, but the nurse should further evaluate the client with this level.

A nurse is caring for a client with a chest tube connected to a three-chamber drainage system without suction. On the illustration below, identify which chamber the nurse will mark to record the current drainage level.

A chest tube drains blood, fluid, and air from around the lungs. The drainage system, which the nurse measures each shift, is on the right. It has three calibrated chambers that show the amount of drainage collected. When the first chamber fills, drainage empties into the second; when the second chamber fills, drainage flows into the third. The water seal chamber is located in the center. The suction control chamber is on the left.

The nurse is assessing a client who has been in a car accident. The client reports sore ribs and painful breathing on the left side of the chest wall. A chest X-ray confirms fracture of two ribs and left-sided hemopneumothorax. What can the nurse anticipate? A chest tube will be inserted into the left pleural space and attached to a pleural evacuation device. Oxygen will be initiated and a bronchoscopy will be performed to identify the area of damage. Aspirational thoracentesis will be performed to remove the accumulated bloody fluid. Splinting of the affected ribs will be initiated and limitation of upper body activity recommended.

A chest tube will be inserted into the left pleural space and attached to a pleural evacuation device. Shortness of breath and decreased breath sounds will be present if there is collapse of the lung because of loss of integrity of the pleural space. The chest tube will need to be inserted because of the rib fractures that have resulted in air and blood in the pleural space. The chest tubes will be removed when the hemopneumothorax has resolved. A thoracentesis will not be enough to resolve the hemopneumothorax; splinting of the ribs will not resolve the hemopneumothorax. Oxygen would be indicated, but a bronchoscopy will not confirm the area of damage if the lung is collapsed.

A nurse preparing to administer medications to a client admitted to a respiratory unit is using the computerized medication-dispensing system and finds that the password is not working. What should the nurse do? Select all that apply. Use another nurse's password to finish dispensing the medication. Ask computer support to reset the password. Override the machine and deliver the medications. Give the medications manually, since the client is having respiratory problems. Secure the medication until the problem can be corrected.

Ask computer support to reset the password. Secure the medication until the problem can be corrected.

A client is admitted to the emergency department with a headache, weakness, and slight confusion. The physician diagnoses carbon monoxide poisoning. What should the nurse do first? Obtain a psychiatric referral. Initiate gastric lavage. Administer 100% oxygen by mask. Maintain body temperature.

Administer 100% oxygen by mask. Carbon monoxide poisoning develops when carbon monoxide combines with hemoglobin. Because carbon monoxide combines more readily with hemoglobin than oxygen does, tissue anoxia results. The nurse should administer 100% oxygen by mask to reduce the half-life of carboxyhemoglobin. Gastric lavage is used for ingested poisons. With tissue anoxia, metabolism is diminished, with a subsequent lowering of the body's temperature, thus steps to increase body temperature would be required. Unless the carbon monoxide poisoning is intentional, a psychiatric referral would be inappropriate.

A client with a history of asthma is admitted to the emergency department. The nurse notes that the client is dyspneic, with a respiratory rate of 35 breaths/min, nasal flaring, and use of accessory muscles. Auscultation of the lung fields reveals greatly diminished breath sounds. What should the nurse do first? Administer bronchodilators as prescribed. Initiate oxygen therapy as prescribed, and reassess the client in 10 minutes. Draw blood for an arterial blood gas. Encourage the client to relax and breathe slowly through the mouth.

Administer bronchodilators as prescribed. In an acute asthma attack, diminished or absent breath sounds can be an ominous sign indicating lack of air movement in the lungs and impending respiratory failure. The client requires immediate intervention with inhaled bronchodilators, IV corticosteroids, and, possibly, IV theophylline. Administering oxygen and reassessing the client 10 minutes later would delay needed medical intervention, as would drawing blood for an arterial blood gas analysis. It would be futile to encourage the client to relax and breathe slowly without providing the necessary pharmacologic intervention.

A nurse is assisting a client with a chronic respiratory disease to walk in the hallway. The nurse observes as the client's SpO2 drops from 94% to 88% during ambulation. Which nursing action is appropriate? Notify the healthcare provider. Administer low flow supplemental O2. Encourage the client to continue with walking. Document the SpO2 and continue to monitor.

Administer low flow supplemental O2. The drop in SpO2 to 88% indicates that the client is hypoxemic and needs supplemental oxygen when exercising. The priority action is to administer low flow oxygen to improve the client's oxygenation status. The nurse should also allow the client to rest, check for an order for oxygen and get one if needed, and document the change in client status and interventions. If the oxygenation status does not improve, the nurse should contact the healthcare provider.

The nurse is caring for a client scheduled for a bronchoscopy. Which interventions should the nurse perform to prepare the client for this procedure? Select all that apply. Provide a clear liquid diet for 6 to 12 hours before the test. Withhold food and fluids for 2 hours before the test. Confirm that a signed informed consent form has been obtained. Ask the client to remove any dentures. Administer prescribed atropine and a sedative.

Administer prescribed atropine and a sedative. Confirm that a signed informed consent form has been obtained. Ask the client to remove any dentures. A signed informed consent form is required for all invasive procedures. Although the health care provider is responsible for explaining the procedure and obtaining consent, the nurse may be required to confirm signature of the consent form. Dentures need to be removed for bronchoscopy, because they may become dislodged during the procedure. Atropine is administered before bronchoscopy to decrease secretions. A sedative may be given to relax the client. Food and fluids are restricted for 6 to 12 hours before the test to avoid the risk of aspiration during the procedure.

A physician orders triamcinolone and salmeterol for a client with a history of asthma. What action should the nurse take when administering these drugs? Allow the client to choose the order in which the drugs are administered. Monitor the client's theophylline level before administering the medications. Administer the salmeterol and then administer the triamcinolone. Administer the triamcinolone and then administer the salmeterol.

Administer the salmeterol and then administer the triamcinolone. A client with asthma typically takes bronchodilators and uses corticosteroid inhalers to prevent acute episodes. Triamcinolone is a corticosteroid; Salmeterol is an adrenergic stimulant (bronchodilator). If the client is ordered a bronchodilator and another inhaled medication, the bronchodilator should be administered first to dilate the airways and to enhance the effectiveness of the second medication. The client may not choose the order in which these drugs are administered because they must be administered in a particular order. Monitoring the client's theophylline level isn't necessary before administering these drugs because neither drug contains theophylline.

An elected official has asked the nurse for information on anthrax and the risk it could pose to residents of the community. What information would the provide? Anthrax can infect the integumentary, GI, and respiratory systems. All community members should receive the anthrax vaccine. Helath care providers use isoniazid (INH), rifampin, and pyrazinamide to treat anthrax. The primary mode of transmission is contact with the blood or body secretions of infected individuals.

Anthrax can infect the integumentary, GI, and respiratory systems. Anthrax can infect the integumentary, GI, and respiratory systems. Infection occurs through inhalation of spores and the ingestion of, or contact with, infected animals. Although an anthrax vaccine exists, its very limited supply makes it impractical to immunize entire communities; it use is typically limited to individuals who work with anthrax in the laboratory environment, to those who work closely with animals, and to military personnel at risk for exposure via biological attack. Additionally, its use is limited to adults ages 18 to 65. Isoniazid, rifampin, and pyrazinamide are used to treat tuberculosis, not anthrax. Penicillin is the most common drug used to threat anthrax.

A client has a sucking stab wound to the chest. Which action should the nurse take first? Draw blood for a hematocrit and hemoglobin level. Prepare a chest tube insertion tray. Prepare to start an I.V. line. Apply a dressing over the wound and tape it on three sides.

Apply a dressing over the wound and tape it on three sides. The nurse should immediately apply a dressing over the stab wound and tape it on three sides to allow air to escape and to prevent tension pneumothorax (which is more life-threatening than an open chest wound). Only after covering and taping the wound should the nurse draw blood for laboratory tests, assist with chest tube insertion, and start an I.V. line.

A client has a plural chest tube following removal of the lower lobe of the lung. Two days after surgery, the tube is accidentally pulled out of the chest wall. What should the nurse do first? Instruct the client to cough to expand the lung. Apply an occlusive dressing such as petroleum jelly gauze. Immerse the tube in sterile water. Auscultate the lung to determine whether it collapsed.

Apply an occlusive dressing such as petroleum jelly gauze. If the chest tube is accidentally pulled out (a rare occurrence), a petroleum jelly gauze and sterile 4×4 inch dressing should be applied over the chest wall insertion site immediately. The dressing should be covered with adhesive tape and be occlusive, and the surgeon should be notified. The lungs can be auscultated and vital signs can be taken after the dressing is in place and the surgeon has been called. Placing the tube in sterile water will not reestablish a seal to prevent air entering the insertion site of the chest tube.

After nasal surgery, the client expresses concern about how to decrease facial pain and swelling while recovering at home. Which instruction would be most effective for decreasing pain and edema? Use corticosteroid nasal spray as needed to control symptoms. Use a bedside humidifier while sleeping. Apply cold compresses to the area. Take analgesics every 4 hours around the clock.

Apply cold compresses to the area. Applying cold compresses helps to decrease facial swelling and pain from edema. Analgesics may decrease pain, but they do not decrease edema. A corticosteroid nasal spray would not be administered postoperatively because it can impair healing. Use of a bedside humidifier promotes comfort by providing moisture for nasal mucosa, but it does not decrease edema.

A client with chronic renal failure is being admitted with pulmonary edema. Which is the priority nursing intervention? Obtain daily weight. Assess lung sounds and oxygen saturation level. Monitor serum potassium levels. Record accurate intake and output.

Assess lung sounds and oxygen saturation level. A client with chronic renal failure who is admitted with pulmonary edema should have their lungs monitored as the priority. The client could have hyperkalemia, hypertension, or anemia, and these assessments should be completed as well. Recording accurate intake and output and monitoring daily weight would also be appropriate. However, respiratory status is the priority.

A nurse is caring for a client on mechanical ventilation who's restless and trying to remove the endotracheal (ET) tube. Which action should the nurse perform next? Apply wrist restraints to the client. Turn the lights on in the room so the nurse can observe the client closely. Assess the client for pain and medicate as appropriate. Tell the client they need to leave the ET tube alone or the nurse will have to use restraints.

Assess the client for pain and medicate as appropriate. A client who's restless may be in pain and may not be able to communicate well because of the ET tube. Therefore, the nurse should assess the client and administer medication as appropriate. The nurse shouldn't apply restraints without trying less-restrictive calming measures, which may include decreasing stimulation by turning the light off or diversional activities, such as music or television. It's never appropriate for the nurse to threaten the client with restraints.

A nurse is caring for a client diagnosed with a deep vein thrombosis (DVT). The client begins to experience symptoms of chest pain, dyspnea, and restlessness. Physical assessment reveals a heart rate of 140 beats per minute, blood pressure of 100/60 mm Hg, and respirations of 40 breaths per minute. What is the nurse's priority action? Document the findings and continue to monitor. Order a 12-lead electrocardiogram. Assess the client's oxygen saturation (SaO2) level. Complete a neurological evaluation.

Assess the client's oxygen saturation (SaO2) level. The client has symptoms consistent with a pulmonary embolism (PE). Assessment of airway and breathing and oxygenation status is the priority. Many clients begin treatment for PE on the basis of clinical history, symptoms, and clinical examination before definitive diagnostic testing has been completed. A 12-lead EKG and/or neurological examination will not determine the oxygenation status of the client. Immediate action and nursing interventions are required to stabilize this client and prevent further deterioration.

A client is receiving morphine sulfate by a patient-controlled analgesia (PCA) system after a left lower lobectomy 4 hours ago. The client reports moderately severe pain in the left thorax that worsens when coughing. What should the nurse do first? Request a prescription for a cough suppressant Reassure the client that the PCA system is working and will relieve pain. Encourage the client to take deep breaths and expectorate the mucous that is stimulating the cough. Assess the pain using a pain scale and compare to the previous assessment.

Assess the pain using a pain scale and compare to the previous assessment. Beginning immediately following surgery, the nurse should assess the client for pain frequently and note changes on the pain scale as a guide to pain management. Reassuring the client is not sufficient when the client is reporting pain. The nurse should encourage the client to cough and take deep breaths; cough suppression is contraindicated because the client must raise and expectorate retained secretions.

A client with a tracheostomy tube coughs and dislodges the tracheostomy tube. What should the nurse do first? Attempt reinsertion of tracheostomy tube. Insert the obturator into the stoma to reestablish the airway. Position the client in semi-Fowler's position with the neck hyperextended. Call for emergency assistance.

Attempt reinsertion of tracheostomy tube. The nurse's first action should be to attempt to replace the tracheostomy tube immediately so that the client's airway is reestablished. Although the nurse may also call for assistance, there should be no delay before attempting reinsertion of the tube. The client is placed in a supine position with the neck hyperextended to facilitate reentry of the tube. The obturator is inserted into the replacement tracheostomy tube to guide insertion and is then removed to allow passage of air through the tube.

The client is ready for discharge after surgery for a deviated septum. Which instruction would be appropriate? Apply heat to the nasal area to control swelling. Take aspirin to control nasal discomfort. Avoid brushing the teeth until the nasal packing is removed. Avoid activities that elicit Valsalva's maneuver.

Avoid activities that elicit Valsalva's maneuver. The client should be instructed to avoid any activities that cause Valsalva's maneuver (e.g., straining at stool, vigorous coughing, exercise) to reduce stress on suture lines and bleeding. The client should not take aspirin because of its antiplatelet properties, which may cause bleeding. Oral hygiene is important to rid the mouth of old dried blood and to enhance the client's appetite. Cool compresses, not heat, should be applied to decrease swelling and control discoloration of the area.

A client who has undergone outpatient nasal surgery is ready for discharge and has nasal packing in place. What should the nurse instruct the client to do? Avoid activities that elicit the Valsalva maneuver. Avoid brushing the teeth until the nasal packing is removed. Take aspirin to control nasal discomfort. Apply heat to the nasal area to control swelling.

Avoid activities that elicit the Valsalva maneuver. The client should be instructed to avoid any activities that cause Valsalva's maneuver (e.g., constipation, vigorous coughing, exercise) in order to reduce bleeding and stress on suture lines. The client should not take aspirin because of its antiplatelet properties, which may cause bleeding. Oral hygiene is important to rid the mouth of old dried blood and to enhance the client's appetite. Cool compresses, not heat, should be applied to decrease swelling and control discoloration of the area.

A health care provider (HCP) has just inserted nasal packing for a client with epistaxis. The client is taking ramipril for hypertension. What should the nurse instruct the client to do? Use 81 mg of aspirin daily for relief of discomfort. Remove the packing if there is difficulty swallowing. Omit the next dose of ramipril. Avoid rigorous aerobic exercise.

Avoid rigorous aerobic exercise. Epistaxis, or nosebleed, is a common, sudden emergency. Commonly, no apparent explanation for the bleeding is known. With significant blood loss, systemic symptoms, such as vertigo, increased pulse, shortness of breath, decreased blood pressure, and pallor, will occur. Because aerobic exercise may increase blood pressure and increased blood pressure can cause epistaxis, the client with hypertension should avoid it. Aspirin inhibits platelet aggregation, reducing the ability of the blood to clot. The client should continue to take his antihypertension medication, ramipril. Posterior nasal packing should be left in place for 1 to 3 days.

A client with bacterial pneumonia is coughing up tenacious, purulent sputum. Which action would help liquefy these viscous secretions? Clap and percuss over the affected lung. Perform postural drainage. Perform coughing and deep-breathing exercises. Breathe humidified air.

Breathe humidified air. Humidified air helps to liquefy respiratory secretions, making them easier to raise and expectorate. Postural drainage may be helpful for respiratory hygiene but will not affect the nature of secretions. Vibration and percussion of the chest wall may be helpful for respiratory hygiene but will not affect the nature of secretions. Coughing and deep-breathing exercises may be helpful for respiratory hygiene but will not affect the nature of secretions.

A nurse is caring for a client experiencing an acute asthma attack. The client stops wheezing, and breath sounds aren't audible. What is the likely cause of these assessment findings? Bronchial edema and constriction have worsened. The asthma attack has resolved, and airflow is restored. The administered albuterol (salbutamol) has been effective. The client has developed acute pulmonary edema.

Bronchial edema and constriction have worsened. During an acute asthma attack, wheezing may stop and breath sounds become inaudible because the airways have swollen and gas exchange is limited. If the attack is over and bronchial swelling has decreased, there would be audible breath sounds and no more wheezing. If the administered albuterol was effective, the wheezing would diminish and gas exchange would improve. Pulmonary edema results in bilateral crackles on auscultation.

A client has a chest tube attached to suction. Which interventions would the nurse perform? Select all that apply. Change the dressing as ordered using aseptic technique. Mark the amount of drainage in the chamber at the end of the shift. Clamp the chest tube when suctioning the client. Palpate the surrounding area of the chest tube for crepitus. Position the client on the side to apply pressure to the chest tube site.

Change the dressing as ordered using aseptic technique. Palpate the surrounding area of the chest tube for crepitus. Mark the amount of drainage in the chamber at the end of the shift. Leakage of air into the subcutaneous tissue is evidenced by a crackling sound when the area is gently palpated. This is referred to as crepitus or subcutaneous emphysema. Aseptic technique would be maintained when the dressing change was performed. At the end of each shift, the amount of drainage is marked on the chest tube container. The tubing is never clamped, as clamping could create a tension pneumothorax. It is inappropriate to position the client directly on the chest tube site, as it would create pain and pressure.

When assessing a client with chest trauma, the nurse notes that the client is taking small breaths at first, then bigger breaths, and then a couple of small breaths, then 10 to 20 seconds of no breaths. How should the nurse should record the breathing pattern? hyperventilation Cheyne-Stokes respiration Biot's respiration obstructive sleep apnea

Cheyne-Stokes respiration Cheyne-Stokes respiration is defined as a regular cycle that starts with normal breaths, which increase and then decrease followed by a period of apnea. It can be related to heart failure or a dysfunction of the respiratory center of the brain. Hyperventilation is associated with an increased rate and depth of respirations. Obstructive sleep apnea is recurring episodes of upper airway obstruction and reduced ventilation. Biot's respiration, also known as "cluster breathing," is periods of normal respirations followed by varying periods of apnea.

Which health-promoting activity should the nurse teach the client who recently underwent a laryngectomy? Develop an aggressive program of exercise to increase airway functioning. Dehumidify the air for comfort. Avoid taking tub baths. Cleanse the mouth three times a day.

Cleanse the mouth three times a day. Oral hygiene is an important aspect of self-care for the laryngectomy client, who is less able to detect mouth odor. Additionally, the mouth harbors bacteria. Good mouth care reduces the risk of infection.The client is able to take tub baths with careful instruction on ways to avoid slipping, the need to make sure the water is no more than 6 inches (15 cm) deep, and other safety measures that will prevent water from entering the laryngectomy site.Moderate exercise may be beneficial, but an aggressive exercise program is not usually part of the plan of care.Air should be humidified to enhance comfort.

The nurse is making a follow-up telephone call to a 52-year-old client with lung cancer. The client now has a low-grade fever (100.6°F [38.1°C]), nonproductive cough, and increasing fatigue. The client completed the radiation therapy to the mass in the right lung and mediastinum 10 weeks ago and has a follow-up appointment to see the health care provider in 2 weeks. What should the nurse advise the client to do? Contact the health care provider (HCP) for an appointment today. This is an expected side effect of the radiation therapy and to keep the follow-up appointment in 2 weeks. Take two acetaminophen tablets every 4 to 6 hours for 2 days and call the health care provider (HCP) if the temperature increases to 101°F (38.3°C) or greater. Go to the nearest emergency department.

Contact the health care provider (HCP) for an appointment today. The client is exhibiting early symptoms of pulmonary toxicity as a result of the radiation therapy. These are not expected adverse effects of radiation. The client should be examined to differentiate between an infection and radiation pneumonitis. Suggesting that the client take acetaminophen and call back in 2 days is inappropriate. These signs and symptoms are not indicative of a true emergency, but the client should be seen by a HCP before the next appointment.

A client with deep vein thrombosis suddenly develops dyspnea, tachypnea, and chest discomfort. What should the nurse do first? Encourage the client to cough and deep breathe. Auscultate the lungs to detect abnormal breath sounds. Contact the health care provider (HCP). Elevate the head of the bed 30 to 45 degrees.

Elevate the head of the bed 30 to 45 degrees. Elevating the head of the bed facilitates breathing because the lungs are able to expand as the diaphragm descends. Coughing and deep breathing do not alleviate the symptoms of a pulmonary embolus, nor does lung auscultation. The HCP must be kept informed of changes in a client's status, but the priority in this case is alleviating the symptoms.

A client is in the emergency department with sneezing and coughing. The client is in the triage area, waiting to be seen by a health care provider. To prevent spread of infection to others in the area and to the health care staff, what should the nurse do? Ask the others in the area to move away from the client. Give the client a surgical mask to wear. Place the client in an isolation room. Ask the client to wash the hands before being examined.

Give the client a surgical mask to wear. In order to prevent infections in hospitals, the nurse institutes measures to contain respiratory secretions in symptomatic clients. The nurse gives the client a mask to wear, and tissues; the nurse instructs the client to dispose of used tissues in a no-touch receptacle. It is not necessary to place the client in isolation. It is not appropriate to ask others to move away from the client, but the nurse can ask the client to keep 3 feet away from others in the waiting room, if there is room. The nurse instructs the client to perform hand hygiene after blowing his nose or touching his nose or face, but doing so is not a prerequisite for being examined by the HCP. The nurse and HCP also use hand hygiene practices when caring for this client.

A client who has asthma is taking albuterol to treat bronchospasms. The nurse should assess the client for which of the following adverse effects that can occur as a result of taking this drug? Select all that apply. Headache. Constipation. Lethargy. Nausea. Nervousness.

Headache. Nausea. Nervousness. Albuterol is a beta-adrenergic agonist. Possible adverse effects include nausea, headache, and nervousness as well as insomnia and vomiting. Constipation is not associated with this drug. The client will not become lethargic; instead, he may experience restlessness.

For a client with chronic obstructive pulmonary disease who has trouble raising respiratory secretions, which intervention would help reduce the tenacity of secretions? Ensure that the client's oxygen therapy is continuous. Help the client maintain an adequate fluid intake. Take a diet history to determine if the client's diet is low in salt. Keep the client in a semi-sitting position as much as possible.

Help the client maintain an adequate fluid intake. A fluid intake of 2 to 3 L/day, providing that the client does not have cardiovascular or renal disease, helps liquefy bronchial secretions.A low-salt diet, continuous oxygen therapy, and maintaining a semi-sitting position do not help reduce the viscosity of mucus.

Which information should the nurse include in a teaching plan for the client newly diagnosed with chronic obstructive pulmonary disease (COPD)? Select all that apply. High humidity increases the effort of breathing. Smoking cessation is important to slow or stop disease progression. Pulmonary rehabilitation programs offer very little benefit. Pneumococcal vaccination is contraindicated for clients with lung disease. A bronchodilator with meter-dose inhaler should be readily available.

High humidity increases the effort of breathing. Smoking cessation is important to slow or stop disease progression. A bronchodilator with meter-dose inhaler should be readily available.

A client with chronic obstructive pulmonary disease (COPD) is experiencing dyspnea and has a low PaO2 level. The nurse plans to administer oxygen as ordered. Which statement is true concerning oxygen administration to a client with COPD? High oxygen concentrations may inhibit the hypoxic stimulus to breathe. High oxygen concentrations will cause coughing and dyspnea. Increased oxygen use will cause the client to become dependent on the oxygen. Administration of oxygen is contraindicated in clients who are using bronchodilators.

High oxygen concentrations may inhibit the hypoxic stimulus to breathe. Clients who have a long history of COPD may retain carbon dioxide (CO2). Gradually the body adjusts to the higher CO2 concentration, and the high levels of CO2 no longer stimulate the respiratory center. The major respiratory stimulant then becomes hypoxemia. Administration of high concentrations of oxygen eliminates this respiratory stimulus and leads to hypoventilation. Oxygen can be drying if it is not humidified, but it does not cause coughing and dyspnea. Increased oxygen use will not create an oxygen dependency; clients should receive oxygen as needed. Oxygen is not contraindicated with the use of bronchodilators.

A nurse is caring for a client with chest trauma. Which nursing diagnosis takes the highest priority? Decreased cardiac output Anxiety Ineffective tissue perfusion (cardiopulmonary) Impaired gas exchange

Impaired gas exchange For a client with chest trauma, a diagnosis of Impaired gas exchange takes priority because adequate gas exchange is essential for survival. Although the other nursing diagnoses — Anxiety, Decreased cardiac output, and Ineffective tissue perfusion (cardiopulmonary) — are possible for this client, they are lower priorities than Impaired gas exchange.

A client with an exacerbation of chronic obstructive pulmonary disease (COPD) is admitted to the hospital. Which nursing diagnosis requires the nurse to collaborate with other health team members to achieve the best outcome for the client? Activity intolerance Imbalanced nutrition: Less than body requirements Impaired skin integrity Impaired gas exchange

Impaired gas exchange Impaired gas exchange requires collaboration between the nurse, physician, and respiratory therapist to help achieve the best respiratory outcome for the client. Medications, oxygen, nebulizer treatments, and arterial blood gas analyses all require a physician's order. The respiratory therapist administers the oxygen and nebulizer treatments. The nurse assesses the client's response to medications and respiratory treatments and provides feedback to the physician and respiratory therapist. Impaired skin integrity, Activity intolerance, and Imbalanced nutrition: Less than body requirements (when applied to the client with COPD) require independent nursing interventions without collaboration with other health team members. These interventions include skin care, pacing nursing care to promote rest and minimize fatigue, and providing small, frequent meals.

A nurse is caring for a client after a thoracotomy for a lung mass. Which nursing diagnosis should be the first priority? Impaired airway clearance Ineffective breathing pattern Impaired gas exchange Impaired physical mobility

Impaired gas exchange Impaired gas exchange should be the nurse's first priority because of the lack of ventilation due to the surgical procedure and pain. There is no indication that the airway is not patent, and mobility and breathing pattern are not first priorities.

A nurse is caring for a client who was admitted with pneumonia, has a history of falls, and has skin lesions resulting from scratching. The priority nursing diagnosis for this client should be Ineffective airway clearance. Risk for falls. Impaired tissue integrity. Ineffective breathing pattern.

Ineffective airway clearance. Ineffective airway clearance is the priority nursing diagnosis for this client. Pneumonia involves excess secretions in the respiratory tract and inhibits air flow to the capillary bed. A client with pneumonia may not have an Ineffective breathing pattern, such as tachypnea, bradypnea, or Cheyne-Stokes respirations. Risk for falls and Impaired tissue integrity aren't priority diagnoses for this client.

A client with suspected severe acute respiratory syndrome (SARS) comes to the emergency department. Which physician order should the nurse implement first? Institute isolation precautions. Obtain a sputum specimen for enzyme immunoassay testing. Begin an I.V. infusion of dextrose 5% in half-normal saline solution at 100 ml/hour. Obtain a nasopharyngeal specimen for reverse-transcription polymerase chain reaction testing.

Institute isolation precautions. SARS, a highly contagious viral respiratory illness, is spread by close person-to-person contact. Contained in airborne respiratory droplets, the virus is easily transmitted by touching surfaces and objects contaminated with infectious droplets. The nurse should give top priority to instituting infection-control measures to prevent the spread of infection to emergency department staff and clients. After isolation measures are carried out, the nurse can begin an I.V. infusion of dextrose 5% in half-normal saline and obtain nasopharyngeal and sputum specimens.

A nurse is instructing a client on how to use an incentive spirometer. The nurse should instruct the client to use the spirometer using steps. Place the steps in order from first to last. All options must be used.

Instruct the client to client exhale fully. Instruct the client to inhale on the mouthpiece and hold the breath for 3 seconds. Instruct the client to passively exhale. Instruct the client to take a deep breath and cough. The nurse should instruct the client to first exhale fully. The client should then place the mouthpiece of the spirometer in the mouth and inhale and hold the breath for 3 seconds. The client should then exhale passively. Finally, the client should take a deep breath and cough.

A client with chronic obstructive pulmonary disease (COPD) takes theophylline, 200 mg P.O. twice per day. During a routine clinic visit, the client asks the nurse how the drug works. What is the mechanism of action of theophylline in treating a nonreversible obstructive airway disease such as COPD? It inhibits the enzyme phosphodiesterase, decreasing degradation of cyclic adenosine monophosphate, a bronchodilator. It alters diaphragm movement, increasing chest expansion and enhancing the lung's capacity for gas exchange. It makes the central respiratory center more sensitive to carbon dioxide and stimulates the respiratory drive. It stimulates adenosine receptors, causing bronchodilation.

It makes the central respiratory center more sensitive to carbon dioxide and stimulates the respiratory drive. In emphysema, the anteroposterior diameter of the chest wall is increased. As a result, the client's breath sounds may be diminished. Fine crackles are present when there is fluid in the lungs. Stridor occurs as a result of a partially obstructed larynx or trachea; stridor can be heard without auscultation. A pleural friction rub is present when pleural surfaces are inflamed and rub together.

The nurse is caring for an elderly client with a possible diagnosis of pneumonia who has just been admitted to the hospital. The client is slightly confused and is experiencing difficulty breathing. Which activities would be appropriate for the nurse to delegate to the unlicensed assistive personnel (UAP)? Select all that apply. Keep the client oriented. Assess the client's breath sounds. Obtain vital signs. Initiate oxygen therapy as needed. Apply antiembolic stockings.

Keep the client oriented. Obtain vital signs. Apply antiembolic stockings. It is appropriate for the nurse to delegate obtaining vital signs and applying antiembolic stockings to the UAP. The UAP can also help keep the client oriented to time, person, and place by talking with the client. The registered nurse is responsible for evaluating the quality and character of the client's vital signs, but the assistant may take the vital signs and report readings to the nurse. It is the registered nurse's responsibility to assess the client's need for oxygen therapy and apply as needed in accordance with the health care provider's prescriptions. It is also the registered nurse's responsibility to perform the nursing history and assess the client's breath sounds.

The nurse is planning care for a client 1 day after having surgery to create a tracheostomy. Which nursing action is the priority goal for the client at this time? Keep the secretions moist with saline. Teach the client about tracheostomy care. Relieve anxiety about breathing. Maintain a patent airway.

Maintain a patent airway. The priority for a client with a new tracheostomy is to maintain a patent airway. A new tracheostomy commonly causes bleeding and excess secretions, and the client may require frequent suctioning to maintain a patent airway. The nurse can keep the secretions moist as a part of the suctioning procedure. The nurse can help the client manage anxiety while maintaining the priority of a patent airway. While the nurse will begin teaching the client about self-care, this is not the priority at this time.

Immediately following an automobile accident, a 21-year-old client has severe pain in the right chest from hitting the steering wheel and a compound fracture of the right tibia and fibula and multiple lacerations and contusions. What is the priority nursing goal for this client? Reduce the client's anxiety. Maintain adequate circulating volume. Decrease chest pain. Maintain adequate oxygenation.

Maintain adequate oxygenation. Blunt chest trauma can lead to respiratory failure. Maintenance of adequate oxygenation is the priority for the client. Decreasing the client's anxiety is related to maintaining effective respirations and oxygenation. Although pain is distressing to the client and can increase anxiety and decrease respiratory effectiveness, pain control is secondary to maintaining oxygenation, as is maintaining adequate circulatory volume.

A client who has been diagnosed with tuberculosis has been placed on drug therapy. The medication regimen includes rifampin. Which instruction should the nurse give the client about potential adverse effects of rifampin? Select all that apply. Maintain follow-up monitoring of liver enzymes. Avoid alcohol intake. The urine may have an orange color. Decrease protein intake in the diet. Have eye examinations every 6 months.

Maintain follow-up monitoring of liver enzymes. Avoid alcohol intake. The urine may have an orange color. A potential adverse effect of rifampin is hepatotoxicity. Clients should be instructed to avoid alcohol intake while taking rifampin and keep follow-up appointments for periodic monitoring of liver enzyme levels to detect liver toxicity. Rifampin causes the urine to turn an orange color, and the client should understand that this is normal. It is not necessary to restrict protein intake in the diet or have the eyes examined due to rifampin therapy.

The nurse team leader is making rounds and observes the client who had a tracheostomy tube inserted 2 days ago (see figure). The nursing policy manual recommends use of the gauze pad. What should the nurse do? Make sure the gauze pad is dry and the client is in a comfortable position. Reposition the gauze pad around the stoma with the open end downward. Ask the unlicensed assistive personnel to tie the tracheostomy tube ties in the back of the client's neck. Ask a registered nurse to change the ties and position another gauze pad around the stoma.

Make sure the gauze pad is dry and the client is in a comfortable position. The tracheostomy tube, ties, and gauze pad are positioned correctly; the nurse team leader should be sure the client is comfortable. The tracheostomy tube ties should be tied in a square knot on the side of the neck and alternate sides of the neck when the ties are changed. The full part of the gauze square should be placed under the tracheostomy tube to absorb drainage. There is no indication the ties need to be changed; an additional gauze pad is not necessary; if necessary, the current gauze square should be changed rather than add an additional pad.

A client with suspected inhalation anthrax is admitted to the emergency department. Which action by the nurse takes the highest priority? Assess intake and output and maintain adequate hydration. Suction the client as needed to obtain a sputum specimen for culture and sensitivity. Reassure the client that intubation and mechanical ventilation will be temporary. Monitor vital signs and oxygen saturation every 15 to 30 minutes.

Monitor vital signs and oxygen saturation every 15 to 30 minutes. Monitoring vital signs and oxygen saturation every 15 to 30 minutes takes priority. Suctioning the client as needed to obtain a sputum specimen may be necessary, but assessing the client for changes in their respiratory status takes priority. Assessing intake and output and providing adequate hydration are important steps for liquefying secretions; however, they don't take priority. Reassuring the client that intubation and mechanical ventilation is temporary is inappropriate. The client may not require intubation and mechanical ventilation; however, if they do, the nurse can't predict the length of time it may be necessary.

What is the rationale that supports multidrug treatment for clients with tuberculosis? Multiple drugs allow reduced drug dosages to be given. Multiple drugs potentiate the drugs' actions. Multiple drugs reduce undesirable drug adverse effects. Multiple drugs reduce development of resistant strains of the bacteria.

Multiple drugs reduce development of resistant strains of the bacteria. Use of a combination of antituberculosis drugs slows the rate at which organisms develop drug resistance. Combination therapy also appears to be more effective than single-drug therapy. Many drugs potentiate (or inhibit) the actions of other drugs; however, this is not the rationale for using multiple drugs to treat tuberculosis. Treatment with multiple drugs does not reduce adverse effects and may expose the client to more adverse effects. Combination therapy may allow some medications (e.g., antihypertensives) to be given in reduced dosages; however, reduced dosages are not prescribed for antibiotics and antituberculosis drugs.

The nurse is assessing the client (see photo) who has recently returned from a 2-month mission in Africa. What type of respiratory protection is appropriate for the staff? double-layered surgical mask no respiratory protection needed N95 particulate respirator surgical mask with eye shield

N95 particulate respirator Any type of blistering lesion, such as smallpox, requires extreme care to prevent exposure. Transmission-based precautions for smallpox includes airborne, droplet, and contact precautions. The N95 mask filters at least 95% of airborne particles. To prevent exposure through the respiratory tract, the N95 mask must be fitted and worn properly.

A client with chronic obstructive pulmonary disease presents with respiratory acidosis and hypoxemia. The client tells the nurse that they don't want to be placed on a ventilator. What action should the nurse take? Notify the physician immediately to have the physician determine client competency. Consult the palliative care group to direct care for the client. Determine whether the client's family was consulted about this decision. Have the client sign a do-not-resuscitate (DNR) form.

Notify the physician immediately to have the physician determine client competency. Three requirements are necessary for informed decision-making: the decision must be given voluntarily; the client making the decision must have the capacity and competence to understand; and the client must be given adequate information to make the decision. In light of the client's respiratory acidosis and hypoxemia, the client might not be competent to make this decision. The physician should be notified immediately so the physician can determine client competency. The physician, not the nurse, is responsible for discussing the implications of a DNR order with the client. The Patient's Bill of Rights entitles the client to make decisions about the care plan, including the right to refuse recommended treatment. The client's family may oppose the client's decision. Consulting the palliative care group isn't appropriate at this time and must be initiated by a physician order.

A client is using an over-the-counter nasal spray containing pseudoephedrine to treat allergic rhinitis. Which instruction about this medication would be most appropriate for the nurse to provide for the client? Prolonged use of nasal spray can lead to nasal infections. Pseudoephedrine is an addictive drug and must be used cautiously. A common side effect of pseudoephedrine nasal spray is thrush. Overuse of pseudoephedrine can lead to increased nasal congestion.

Overuse of pseudoephedrine can lead to increased nasal congestion. Overuse of nasal spray containing pseudoephedrine can lead to rhinitis medicamentosa, which is a rebound effect causing increased swelling and congestion. Use of pseudoephedrine nasal spray does not cause infections or thrush. Pseudoephedrine is not addictive.

A client who underwent a left lower lobectomy has been out of surgery for 48 hours. The client is receiving morphine sulfate via a patient-controlled analgesia (PCA) system and reports having pain in the left thorax that worsens when coughing. After checking the PCA system, what should the nurse do next? Obtain a more detailed assessment of the client's pain using a pain scale. Let the client rest so the client is not stimulated to cough. Encourage the client to take deep breaths to help control the pain. Reassure the client that the machine is working and will administer medication to relieve the pain.

Obtain a more detailed assessment of the client's pain using a pain scale. Systematic pain assessment is necessary for adequate pain management in the postoperative client. Guidelines from a variety of health care agencies and nursing groups recommend that institutions adopt a pain assessment scale to assist in facilitating pain management. Even though the client is receiving morphine sulfate by PCA, and the pump is working, the nurse should continue to assess the client. The concern is not to eliminate coughing but to control pain adequately. Coughing is necessary to prevent postoperative atelectasis and pneumonia. Breathing exercises may help control pain in some circumstances; however, most clients with thoracic surgery require parenteral opioid analgesics in the early postoperative period. Although it is necessary that the PCA device be checked periodically, reassuring the client is not sufficient, so further assessment is needed.

While making rounds, the nurse finds a client with chronic obstructive pulmonary disease sitting in a wheelchair, slumped over a lunch tray. After determining the client is unresponsive and calling for help, what should the nurse do next? Push the "code blue" (emergency response) button. Call the rapid response team. Call for a defibrillator. Open the client's airway.

Open the client's airway. The nurse has already called for help and established unresponsiveness so the first action is to open the client's airway; opening the airway may result in spontaneous breathing and will help the nurse determine whether or not further intervention is required. Pushing the "code blue" button may not be the appropriate action if the client is breathing and becomes responsive once the airway is open. A quick assessment upon opening the client's airway will help the nurse to determine if the rapid response team is needed. Calling for a defibrillator may not be the necessary or appropriate action once the client's airway has been opened.

The nurse is caring for a client with cystic fibrosis (CF) who has increased dyspnea. Which intervention should the nurse include in the plan of care? Perform chest physiotherapy. Place the client on a fluid restriction. Schedule a sweat chloride test. Suction the client's upper airway.

Perform chest physiotherapy. Airway clearance techniques are treatments that help people with cystic fibrosis (CF) stay healthy and breathe easier. ACTs loosen thick, sticky lung mucus so it can be cleared by coughing. Clearing the airways reduces lung infections and improves lung function. Routine scheduling of airway clearance using chest physiotherapy is an essential intervention for clients with CF. Fluid restrictions will worsen the thickening of secretions and suctioning the upper airway will not reach thick secretions in the lower lungs. A sweat chloride test is used to diagnose CF; it is not a treatment.

The nurse is preparing to suction a tracheostomy for a client with methicillin-resistant Staphylococcus aureus (MRSA) (see figure). What should the nurse do next? Change to a surgical mask. Wear a powered air purifying respirator (PAPR) face shield. Use goggles that include the hairline. Proceed to suction the client's tracheostomy.

Proceed to suction the client's tracheostomy. The nurse is wearing protective personnel equipment appropriate for suctioning the client: goggles, gown, and respirator mask. It is not necessary to wear a PAPR face shield to suction a tracheostomy. A surgical mask does not provide maximum protection.

The client with a laryngectomy is being discharged. The nurse should determine that the client understands the need for which self-care measures? Select all that apply. Follow a low-fiber diet. Use a protective shield over the stoma for bathing. Limit spicy seasonings on food. Provide humidification in the home. Consume a liberal intake of fluids (2 to 3 L/day).

Provide humidification in the home. Use a protective shield over the stoma for bathing. Consume a liberal intake of fluids (2 to 3 L/day). The nurse should advise the client to provide humidification at home. Instruct the client to use a protective shield for bathing, showering, or shampooing or cutting hair to prevent aspiration. The nurse can also encourage the client to obtain a fluid intake of 2 to 3 L daily to help liquefy secretions. To counteract any loss of smell and impairment of taste sensation, the client can add additional seasoning to food. The client should follow a high-fiber diet and use stool softeners because the client may not be able to hold the breath and bear down for bowel movements.

To promote comfort and optimal respiratory expansion for a client with chronic obstructive pulmonary disease during sexual intimacy, the nurse can suggest the couple do what? Raise the affected partner's head and upper torso on pillows. Use a nasal decongestant inhaler. Limit the duration of the sexual activity. Have the affected partner assume a dependent position.

Raise the affected partner's head and upper torso on pillows. Raising the upper torso for the affected partner facilitates respiratory function. The client should not use inhalers that are not a part of the treatment plan, and if the client's health is well managed, it is not necessary to take additional medications to improve respiratory function. A dependent position may compromise respiratory expansion, even though energy may be conserved. Duration of sexual activity is not necessarily related to exertion.

The nurse is planning to teach the client how to properly use a metered-dose inhaler to treat asthma. The nurse should tell the client to use which procedure? Inhale the medication and then exhale through the nose. Rinse the mouth after each use of a steroid inhaler. Inhale quickly when administering the medication. Cough and deep-breathe before inhaling the medication.

Rinse the mouth after each use of a steroid inhaler. Clients should be instructed to rinse their mouths after using a steroid inhaler to avoid developing thrush. Clients should also be instructed to inhale slowly through the mouth and then hold the breath as they count to 10 slowly. It is not necessary for the client to cough and deep-breathe before using the inhaler.

A client with myasthenia gravis is receiving continuous mechanical ventilation. When the high-pressure alarm on the ventilator sounds, what should the nurse do? Increase the oxygen percentage. Ventilate the client with a handheld mechanical ventilator. Check for an apical pulse. Suction the client's artificial airway.

Suction the client's artificial airway. A high-pressure alarm on a continuous mechanical ventilator indicates an obstruction in the flow of oxygen from the machine to the client. The nurse should suction the client's artificial airway to remove respiratory secretions that could be causing the obstruction. The sounding of a ventilator alarm has no relationship to the apical pulse. Increasing the oxygen percentage and ventilating with a handheld mechanical ventilator wouldn't correct the airflow blockage.

A client with tuberculosis is taking isoniazid (INH). What should the nurse instruct the client to do to help prevent development of peripheral neuropathies? Adhere to a low-cholesterol diet. Get extra rest. Supplement the diet with pyridoxine (vitamin B6). Avoid excessive sun exposure.

Supplement the diet with pyridoxine (vitamin B6). INH competes for the available vitamin B6 in the body and leaves the client at risk for development of neuropathies related to vitamin deficiency. Supplemental vitamin B6 is routinely prescribed. Following a low-cholesterol diet, getting extra rest, and avoiding excessive sun exposure will not prevent the development of peripheral neuropathies.

The nurse should include which instructions in the teaching plan for a client with chronic sinusitis? Select all that apply. Take a hot shower in the morning and evening. Limit fluid intake to 1,000 ml per 24 hours. Report a temperature of 102° F (38.9° C) or higher. Perform postural drainage every day. Avoid the use of caffeinated beverages.

Take a hot shower in the morning and evening. Report a temperature of 102° F (38.9° C) or higher. The client with chronic sinusitis should be instructed to take hot showers in the morning and evening to promote drainage of secretions. There is no need to limit caffeine intake. Performing postural drainage will inhibit removal of secretions, not promote it. Clients should elevate the head of the bed to promote drainage. Clients should report all temperatures higher than 100.4° F (38° C) because a temperature that high can indicate infection. The client should increase, not limit fluid intake; a 24 hour fluid intake of 2,000 to 3,000 ml would be appropriate.

Which nursing action would most likely be successful in reducing pleuritic chest pain in a client with pneumonia? Encourage the client to breathe shallowly. Have the client practice abdominal breathing. Offer the client incentive spirometry. Teach the client to splint the rib cage when coughing.

Teach the client to splint the rib cage when coughing. The pleuritic pain is triggered by chest movement and is particularly severe during coughing. Splinting the chest wall will help reduce the discomfort of coughing. Deep breathing is essential to prevent further atelectasis. Abdominal breathing is not as effective in decreasing pleuritic chest pain as is splinting of the rib cage. Incentive spirometry facilitates effective deep breathing but does not decrease pleuritic chest pain.

The nurse is caring for a client preoperatively for a lung tumor removal. Which interventions should the nurse implement? Select all that apply. Teach the client to use the incentive spirometer. Protect the I.V. line to prevent dislodgement during isometric leg exercises. Encourage the client to splint chest with deep breathing and coughing. Administer subcutaneous heparin to prevent deep vein thrombosis. Keep the client in a side lying position to protect the airway.

Teach the client to use the incentive spirometer. Encourage the client to splint chest with deep breathing and coughing. During the preoperative period the nurse will focus on teaching the client to use the incentive spirometer and encouraging the client to splint chest with deep breathing and coughing. Keeping the client side lying, protecting the I.V. line to prevent dislodgement during isometric leg exercises, and administering subcutaneous heparin to prevent deep vein thrombosis are part of postoperative care.

The nurse teaches the client how to instill nose drops. Which technique is correct? The client blows the nose gently before instilling drops. The client uses sterile technique when handling the dropper. The client sits in a semi-Fowler's position for 2 minutes. The client uses a new dropper for each instillation.

The client blows the nose gently before instilling drops. The client should blow the nose before instilling nose drops. Instilling nose drops is a clean technique. The dropper should be cleaned after each administration, but it does not need to be changed. The client should assume a position that will allow the medication to reach the desired area; this is usually a supine position.

A nurse is conducting an initial assessment on a client with possible tuberculosis. Which assessment finding indicates a risk factor for tuberculosis? The client sees their physician for a check-up yearly. The client works in a healthcare insurance office. The client has never traveled outside of the country. The client had a liver transplant 2 years ago.

The client had a liver transplant 2 years ago. A history of immunocompromised status, such as that which occurs with liver transplantation, places the client at a higher risk for contracting tuberculosis. Other risk factors include inadequate healthcare, traveling to countries with high rates of tuberculosis (such as southeastern Asia, Africa, and Latin America), being a healthcare worker who performs procedures in which exposure to respiratory secretions is likely, and being institutionalized.

When attempting to check the pupils of a client scheduled to receive general anesthesia, the nurse notices that the client has trouble tilting the head back. What is the primary concern related to this finding? The ability to assess the client's pupils is limited. The client is at risk for difficult intubation. The client has limited movement of the neck. The client may have postoperative neck pain.

The client is at risk for difficult intubation. The client is at risk for a difficult intubation because the neck must be hyperextended to pass the endotracheal tube. Assessment of the pupils should not be limited. If the client is positioned appropriately during surgery, there is no risk of postoperative neck pain or limited neck movement.

A nurse is caring for a client with a wound infection who develops septic shock. The nurse notes the following arterial blood gas results: pH of 7.25, PaCO2 of 43 mm Hg (5.7 kPa), partial pressure of arterial oxygen (PaO2) of 70 mm Hg (9.3 kPa), and bicarbonate (HCO3-) of 18 mEq/L (18 mmol/L). According to the oxyhemoglobin dissociation curve, the nurse would be most correct to highlight which statement on the shift report? The client's oxyhemoglobin curve is shifted to the left. The client's hemoglobin saturation is close to 100%. The client's hemoglobin saturation is close to 85%. The client's profile reflects alkalosis.

The client's hemoglobin saturation is close to 85%. The oxyhemoglobin dissociative curve describes the tendency for oxygen to bind to hemoglobin. The acidic condition of the blood shifts the oxyhemoglobin dissociation curve to the right. This enables oxygen molecules to unload more easily from the hemoglobin. According to the client's PaO2 value of 70 mm Hg (9.3 kPa) and pH value of 7.25, the hemoglobin saturation is close to 85%.

During inspiration, which action occurs? The diaphragm descends. The lungs recoil. Alveolar pressure is positive. The inspiratory muscles relax.

The diaphragm descends. During inspiration, inspiratory muscles contract, the diaphragm descends, alveolar pressure is negative, and air moves into the lungs. The lungs recoil during expiration.

The nurse is preparing the client diagnosed with pleural effusion for a left-sided thoracentesis. The X-ray shows fluid in the pleural cavity. During the preparation for the procedure, the client asks where the health care provider (HCP) will "put the needle." Select the appropriate site from the diagram.

The fluid typically localizes at the base of the thorax.

A client with an endotracheal tube is being weaned from the ventilator. For which reason should the procedure be terminated? The diastolic blood pressure decreased 6 mm Hg. The client lifts head independently off the pillow. The client is awake and alert. The heart rate increased 20 beats/minute.

The heart rate increased 20 beats/minute. The weaning process should be terminated if the client experiences an adverse reaction such as increase in heart rate of 20 beats/minute. The client being awake and alert and able to lift the head independently off the pillow indicates enough muscle strength and tone to maintain independent respirations. A diastolic blood pressure decrease of 6 mm Hg is not a reason to discontinue the weaning procedure.

A client has a chest tube attached to a water seal drainage system, and the nurse notes that the fluid in the chest tube and in the water seal column has stopped fluctuating. How should the nurse interpret this finding? The mediastinal space has decreased. The chest tube is in the pleural space. The lung has collapsed. The lung has fully expanded.

The lung has fully expanded. Cessation of fluid fluctuation in the tubing can mean one of several things: the lung has fully expanded and negative intrapleural pressure has been reestablished; the chest tube is occluded; or the chest tube is not in the pleural space. Fluid fluctuation occurs because during inspiration intrapleural pressure exceeds the negative pressure generated in the water-seal system. Therefore, drainage moves toward the client. During expiration, the pleural pressure exceeds that generated in the water-seal system, and fluid moves away from the client. When the lung is collapsed or the chest tube is in the pleural space, fluid fluctuation is likely to be noted. The chest tube is not inserted in the mediastinal space.

A client with acute bronchitis is admitted to the healthcare facility and is receiving supplemental oxygen via nasal cannula. When monitoring this client, the nurse suddenly hears a high-pitched whistling sound. What is the most likely cause of this sound? The oxygen tubing is pinched. The client has a nasal obstruction. The water level in the humidifier reservoir is too low. The oxygen concentration is above 44%.

The oxygen tubing is pinched. Pinching of the tubing used to deliver oxygen causes a high-pitched whistling sound. When the water level in the humidifier reservoir is too low, the oxygen tubing appears dry but doesn't make noise. A client with a nasal obstruction becomes more uncomfortable with nasal prongs in place and doesn't experience relief from oxygen therapy; the client's complaints, not an abnormal sound, would alert the nurse to this problem. A nasal cannula can't deliver oxygen concentrations above 44%.

A nurse is assessing a client using a tracheostomy tube. The client has bilateral rhonchi in the upper lobes of the lungs and is unsuccessful in coughing up secretions. Which action should the nurse take? Call respiratory therapy for a breathing treatment. Use a sterile suction kit to suction the client. Encourage the use of the incentive spirometer. Teach the client pursed lip breathing.

Use a sterile suction kit to suction the client. The priority for this client is suctioning to remove secretions in the upper airway if the client is unable to cough adequately. The other interventions will not effectively assist the client to maintain a patent airway.

The nurse instills 5 mL of normal saline before suctioning a client's tracheostomy tube. Which indicates the instillation is effective? There is humidification for the respiratory tract. The client coughs. There is minimal friction when the catheter is passed into the tracheostomy tube. The secretions are thinned.

The secretions are thinned. The primary purpose of instilling 5 mL of normal saline solution before suctioning a tracheostomy tube is to thin the secretions to be suctioned. The saline may stimulate a cough; however, this is not the reason for using saline. The tracheostomy tube is larger than the suction catheter, so the catheter will easily pass into the tube without lubrication. Humidification is provided by a nebulizer if needed.

A nurse observes constant bubbling in the water-seal chamber of a closed chest drainage system. What should the nurse conclude? The chest tube is obstructed. The system has an air leak. The system is functioning normally. The client has a pneumothorax.

The system has an air leak. Constant bubbling in the water-seal chamber indicates a system air leak and requires immediate intervention. The client with a pneumothorax will have intermittent bubbling in the water-seal chamber. Clients without a pneumothorax should have no evidence of bubbling in the chamber. If the tube is obstructed, the fluid would stop fluctuating in the water-seal chamber.

A nurse is helping a suspected choking victim. When should the nurse perform the Heimlich maneuver? The victim is coughing vigorously. The victim can make only minimal vocal noises. The victim starts to become cyanotic. The victim cannot speak due to airway obstruction.

The victim cannot speak due to airway obstruction. The Heimlich maneuver should be administered only to a client who cannot make any sounds due to airway obstruction. If the client can whisper words or cough, some air exchange is occurring, and the emergency medical system should be called instead of attempting the Heimlich maneuver. Cyanosis may accompany or follow choking; however, the Heimlich maneuver should only be initiated when the client cannot speak.

Which action will be most helpful to the nurse when determining the need for oxygen therapy in a client with chronic obstructive pulmonary disease? Assess the client's fatigue level. Evaluate the client's hemoglobin level daily. Use a pulse oximeter to determine oxygen saturation. Ask the client to tell the nurse when oxygen is needed.

Use a pulse oximeter to determine oxygen saturation. A pulse oximeter, which measures oxygen saturation, is the most effective noninvasive way to determine a client's need for oxygen therapy.Although the client may feel the need for oxygen during periods of dyspnea, this is not a reliable way of determining the client's need.Fatigue may be due to other factors besides oxygenation levels.Evaluating the client's hemoglobin level can provide an indication that the client may have less oxygen-carrying capacity but is not a reliable indicator of oxygen need.

What should the nurse do when suctioning a client who has a tracheostomy tube 3 days following insertion? Protect the catheter in sterile packaging between suctioning episodes. Use a clean catheter with each suctioning, and disinfect it in hydrogen peroxide between uses. Clean the catheter in sterile water after each use, and reuse for no longer than 8 hours. Use a sterile catheter each time the client is suctioned.

Use a sterile catheter each time the client is suctioned. The recommended technique is to use a sterile catheter each time the client is suctioned. There is a danger of introducing organisms into the respiratory tract when strict aseptic technique is not used. Reusing a suction catheter is not consistent with aseptic technique. The nurse does not use a clean catheter when suctioning a tracheostomy or a laryngectomy; it is a sterile procedure.

A nurse is teaching a client with chronic bronchitis about breathing exercises. Which instruction should the nurse include in the teaching? Use chest breathing. Exhale through an open mouth. Make inhalation longer than exhalation. Use diaphragmatic breathing.

Use diaphragmatic breathing. In chronic bronchitis the diaphragm is flat and weak. Diaphragmatic breathing helps to strengthen the diaphragm and maximizes ventilation. Exhalation should be longer than inhalation to prevent collapse of the bronchioles. The client with chronic bronchitis should exhale through pursed lips to prolong exhalation, keep the bronchioles from collapsing, and prevent air trapping. Diaphragmatic breathing — not chest breathing — increases lung expansion.

Which choice demonstrates best nursing practice when performing tracheostomy care on a client who is 8 hours post new insertion? Place the client in the semi-Fowler's position. Monitor the client's temperature after the procedure. Use povidone-iodine to clean the inner cannula when it is removed. Use sterile gloves during the procedure.

Use sterile gloves during the procedure. The tracheotomy site is a portal of entry for microorganisms. Sterile technique must be used within the first 24-48 hours because the site is a new source of infection. Monitoring the client's temperature is not reflected in application of this question. Povidone-iodine destroys new cellular growth, so it is not used on open wounds. The client should be in high Fowler's, not semi-Fowler's position.

A client is receiving streptomycin for the treatment of tuberculosis. The nurse should assess the client for eighth cranial nerve damage by observing the client for: Impaired vision. Facial paralysis. Difficulty swallowing. Vertigo.

Vertigo. The eighth cranial nerve is the vestibulocochlear nerve, which is responsible for hearing and equilibrium. Streptomycin can damage this nerve (ototoxicity). Symptoms of ototoxicity include vertigo, tinnitus, hearing loss, and ataxia. Facial paralysis would result from damage to the facial nerve (VII). Impaired vision would result from damage to the optic (II), oculomotor (III), or the trochlear (IV) nerves. Difficulty swallowing would result from damage to the glossopharyngeal (IX) or the vagus (X) nerve.

The nurse is developing a care plan for a client with tuberculosis. Which measures would be implemented for staff prior to entering the room? Wear a gown and gloves when in contact with the client. Prevent visitors from visiting to reduce the possibility of transmission. Wear a mask, gown, and gloves when providing care. Wear a mask at all times when entering the room.

Wear a mask at all times when entering the room. A special mask that prevents the passage of airborne droplets is needed to intercept airborne droplet transmission. This choice represents the use of transmission-based precautions specifically to prevent the spread of tuberculosis. Gowns and gloves are not required; visitors are welcomed providing they wear a mask during their visit to protect themselves.

A client has just undergone a bronchoscopy. Which nursing interventions are appropriate after this procedure? Select all that apply. Provide sips of water to moisten the client's mouth. Alert the client to resume food and fluids when the client's voice returns. Withhold food and fluids until the client's gag reflex returns. Monitor the client's vital signs. Assess for hemoptysis and frank bleeding. Keep the client flat for at least 2 hours.

Withhold food and fluids until the client's gag reflex returns. Monitor the client's vital signs. Assess for hemoptysis and frank bleeding. To prevent aspiration, the client should not receive food or fluids until the gag reflex returns. Although a small amount of blood in the sputum is expected if a biopsy was performed, frank bleeding indicates hemorrhage and should be reported to the physician immediately. Vital signs should be monitored after the procedure, because a vasovagal response may cause bradycardia, laryngospasm can affect respirations, and fever may develop within 24 hours of the procedure. To reduce the risk of aspiration, the client should be placed in a semi-Fowler's or side-lying position after the procedure until the gag reflex returns. The client does not lose the voice after a bronchoscopy, so voice should not be used as a gauge for resuming food and fluid intake.

During morning assessment, a nurse assesses four clients. Which client is the priority for follow up? a 73-year-old client who has pneumonia with coarse crackles, is receiving 2 L/minute of oxygen, and has an I.V. line an 84-year-old client with heart failure who's on telemetry and 2 L/minute of oxygen a 42-year-old client who has left lower lobe pneumonia and an I.V. line a 48-year-old client with chronic obstructive pulmonary disease with occasional atrial fibrillation

a 73-year-old client who has pneumonia with coarse crackles, is receiving 2 L/minute of oxygen, and has an I.V. line The 73-year-old client with pneumonia should be the nurse's priority because of the oxygenation complications and the audible crackles that may result from fluid overload from the I.V. line. The 42-year-old client is younger and more mobile than the others. The 84-year-old client doesn't have pressing needs at this time. The nurse should evaluate the 48-year-old client if the client goes into atrial fibrillation, but this client isn't a priority at this time.

A nurse is caring for a group of clients on a medical-surgical floor. Which client is at greatest risk for developing pneumonia? a client who has an order for acetaminophen with codeine for pain but has not requested it a client with a history of smoking two packs of cigarettes per day until quitting 2 years ago a client with a nasogastric tube a client who ambulates in the hallway daily

a client with a nasogastric tube Nasogastric, orogastric, and endotracheal tubes increase the risk of pneumonia because of the risk of aspiration from improperly placed tubes. Frequent oral hygiene and checking tube placement help prevent aspiration and pneumonia. Although a client who smokes is at increased risk for pneumonia, the risk decreases if the client has stopped smoking. Ambulation helps prevent pneumonia. A client who receives opioids, not acetaminophen, has a risk of developing pneumonia because respiratory depression may occur.

During the insertion of a rigid scope for bronchoscopy, a client experiences a vasovagal response. The nurse should expect a drop in the client's heart rate. the client's pupils to become dilated. the client to experience bronchodilation. a decrease in the client's gastric secretions.

a drop in the client's heart rate. During a bronchoscopy, a vasovagal response may be caused by stimulating the pharynx, and it, in turn, may cause stimulation of the vagus nerve. The client may, therefore, experience a sudden drop in heart rate leading to syncope. Stimulation of the vagus nerve doesn't lead to pupillary dilation or bronchodilation. Stimulation of the vagus nerve increases gastric secretions.

Which registered nurse should be assigned to the client who had a chest tube inserted yesterday? a registered nurse who worked as a head nurse on the orthopedic unit a charge nurse pulled from the psychiatric unit a registered nurse who use to work on the cardiovascular unit a licensed practical nurse with 10 years of experience

a registered nurse who use to work on the cardiovascular unit According to the National Council of State Boards of Nursing, delegation encompasses five rights — the right task, right circumstance, right person, right direction and communication, and right supervision and evaluation. A registered nurse is the most appropriate caregiver to assign to the client with a chest tube because the chest tube system requires frequent assessment and monitoring. In addition, the client may require immediate nursing intervention should the chest tube became obstructed or dislodged, so a registered nurse with experience on the cardiovascular unit is the most appropriate caregiver. It is not necessary for a charge nurse to care for this client.

A client with a clamped chest tube in place has become increasingly short of breath throughout the shift and reports pain to the right chest wall. The nurse understands that the most likely cause is: an increase of blood in the pleural space. a decrease in pressure in the tube. a bilateral pneumothorax. a tension pneumothorax.

a tension pneumothorax. Atmospheric pressure is greater than the pressure inside the pleural space. If a chest tube were clamped for a period of time, the intrathoracic pressure would increase, and subsequently so would tension. The other choices are not reflective of a clamped chest tube.

An older adult client is admitted to the hospital with a diagnosis of bacterial pneumonia. While obtaining the client's health history, the nurse learns that the client has osteoarthritis, follows a vegetarian diet, and is very concerned with cleanliness. Which client information would most likely be a predisposing factor for the diagnosis of pneumonia? daily bathing vegetarian diet osteoarthritis age

age The client's age is a predisposing factor for pneumonia; pneumonia is more common in older or debilitated clients. Other predisposing factors include smoking, upper respiratory tract infections, malnutrition, immunosuppression, and the presence of a chronic illness. Osteoarthritis, a nutritionally sound vegetarian diet, and frequent bathing are not predisposing factors for pneumonia.

Which statement indicates that the client with chronic obstructive pulmonary disease (COPD) who has been discharged to home understands the care plan? The client: will avoid direct contact with family and friends. will use oxygen via a nasal cannula at 5 L/min. can state actions to reduce pain. agrees to call the health care provider (HCP) if dyspnea on exertion increases.

agrees to call the health care provider (HCP) if dyspnea on exertion increases. Increasing dyspnea on exertion indicates that the client may be experiencing complications of COPD. Therefore, the client should notify the HCP. It is not necessary to avoid being around others. Pain is not a common symptom of COPD. Clients with COPD use low-flow oxygen supplementation (1 to 2 L/min) to avoid suppressing the respiratory drive, which, for these clients, is stimulated by hypoxia.

A client with severe shortness of breath comes to the emergency department. The client tells the emergency department staff that they recently traveled to China for business. Based on the client's travel history and presentation, the staff suspects severe acute respiratory syndrome (SARS). Which isolation precautions should the staff institute? contact precautions contact and droplet precautions airborne and contact precautions droplet precautions

airborne and contact precautions SARS, a highly contagious viral respiratory illness, is spread by close person-to-person contact. The client should be placed on airborne and contact precautions to prevent the spread of infection. Droplet precautions don't require a negative air pressure room and wouldn't protect the nurse who touches contaminated items in the client's room. Contact precautions alone don't provide adequate protection from airborne particles.

The nurse should use which type of precautions for a client being admitted to the hospital with suspected tuberculosis? airborne precautions contact precautions standard precautions droplet precautions

airborne precautions Airborne precautions prevent transmission of infectious agents that remain infectious over long distances when suspended in the air (e.g., Mycobacterium tuberculosis, measles, varicella virus [chickenpox], and possibly SARS-CoV). The preferred placement is in an isolation single-client room that is equipped with special air handling and ventilation. A negative pressure room, or an area that exhausts room air directly outside or through HEPA filters, should be used if recirculation is unavoidable. While standard precautions such as hand hygiene and wearing gloves and a gown is important, they are not sufficient to prevent transmission of tuberculosis. Contact precautions are used with clients with known or suspected infections who present an increased risk for contact transmission. Droplet precautions are intended to prevent transmission of pathogens spread through close respiratory or mucous membrane contact with respiratory secretions. Because these pathogens do not remain infectious over long distances in a health care facility, special air handling and ventilation are not required to prevent droplet transmission.

A severe acute respiratory syndrome (SARS) epidemic is suspected in a community of 10,000 people. As clients with SARS are admitted to the hospital, what type of precautions should the nurse institute? enteric precautions reverse isolation airborne precautions hand-washing precautions

airborne precautions Transmission of SARS can be contained by airborne precautions that include an insolation room with negative pressure, use of N-95 respirator, and use of personal protective equipment. The disease is spread by the respiratory, not enteric, route. Hand washing alone is not sufficient to prevent transmission. Reverse isolation (protection of the client) is not sufficient to prevent transmission.

A 6-year-old child is admitted to a pediatric unit for the treatment of osteomyelitis. Which essential medication classification would the nurse anticipate as documented on the medication report? antibiotic analgesic antipyretic anti-inflammatory

antibiotic Staphylococcus. aureus is the most common causative pathogen of osteomyelitis; the usual source of the infection is an upper respiratory infection (URI) or skin lesion. The nurse anticipates an intravenous antibiotic as the essential medication. The nurse may have an anti-inflammatory medication as adjunct therapy. By decreasing the infection, the client may experience decreased pain; thus, not needing an analgesic. The nurse would administer an antipyretic if the child was febrile.

A client with unresolved hemothorax is febrile, with chills and sweating. The client has a nonproductive cough and chest pain. The chest tube drainage is turbid. What should the nurse request in SBAR communication with the health care provider? portable chest X-ray intubation and mechanical ventilation antibiotic therapy arterial blood gasses

antibiotic therapy Any condition that produces fluid accumulation or sequestration of fluid with infective properties can lead to empyema, an accumulation of pus in a body cavity, especially the pleural space, as a result of bacterial infection. An infected chest tube site, lobar pneumonia, and P. carinii pneumonia can lead to fever, chills, and sweating associated with infection. With the symptoms of infection, antibiotic therapy would be recommended. Nothing in the question demonstrates a need for chest X-ray, intubation, or ABGs.

The nurse is caring for a client diagnosed with chronic obstructive pulmonary disease (COPD) and experiencing respiratory acidosis. The decrease in pH exists because the client's lungs: are unable to blow off carbon dioxide. are unable to exchange oxygen and carbon dioxide. are unable to inspire sufficient oxygen. have ineffective cilia from years of smoking.

are unable to blow off carbon dioxide. In clients with chronic respiratory acidosis, the client is unable to blow off carbon dioxide leaving in increased amount of hydrogen in the system. The increase in hydrogen ions leads to acidosis. In COPD, the client is able to breathe in oxygen and gas exchange can occur, but the lungs' ability to remove carbon dioxide from the system is compromised. Although individuals with COPD frequently have a history of smoking, impaired ciliary function is not the cause of the acidosis.

The nurse is caring for a client with bacterial pneumonia. The effectiveness of the client's oxygen therapy can be best determined by which indicator of oxygenation? arterial blood gas (ABG) values. client's respiratory rate. client's level of consciousness. absence of cyanosis.

arterial blood gas (ABG) values. The client's ABG levels are the most sensitive indicator of the effectiveness of the client's oxygen therapy. Cyanosis is a late sign of decreased oxygenation and is not a reliable indicator. The client's respiratory rate and level of consciousness may be altered because of other problems not related to the client's oxygenation.

A client with chronic obstructive pulmonary disease (COPD) is recovering from a myocardial infarction. Because the client is extremely weak and can't produce an effective cough, the nurse should monitor closely for pleural effusion. oxygen toxicity. pulmonary edema. atelectasis.

atelectasis. In a client with COPD, an ineffective cough impedes secretion removal. This, in turn, causes mucus plugging, which leads to localized airway obstruction — a known cause of atelectasis. An ineffective cough doesn't cause pleural effusion (fluid accumulation in the pleural space). Pulmonary edema usually results from left-sided heart failure, not an ineffective cough. Although many noncardiac conditions may cause pulmonary edema, an ineffective cough isn't one of them. Oxygen toxicity results from prolonged administration of high oxygen concentrations, not an ineffective cough.

A nurse prepares to perform postural drainage. How should the nurse ascertain the best position to facilitate clearing the lungs? chest X-ray inspection arterial blood gas (ABG) levels auscultation

auscultation The nurse should assess breath sounds before doing postural drainage to determine the areas that need draining. Inspection, chest X-rays, and ABG levels are all assessment parameters that give good information about respiratory function but aren't necessary to determine lung areas requiring postural drainage.

A client comes to the emergency department complaining of sudden onset of diarrhea, anorexia, malaise, cough, headache, and recurrent chills. Based on the client's history and physical findings, the physician suspects legionnaires' disease. While awaiting diagnostic test results, the client is admitted to the facility and started on antibiotic therapy. What's the drug of choice for treating legionnaires' disease? amantadine amphotericin B azithromycin rifampin

azithromycin Azithromycin is the drug of choice for treating legionnaires' disease. Rifampin is used to treat tuberculosis. Amantadine, an antiviral agent, and amphotericin B, an antifungal agent, are ineffective against legionnaires' disease, which is caused by bacterial infection.

Before weaning a client from a ventilator, which assessment parameter is the most important for the nurse to obtain? baseline arterial blood gas (ABG) levels prior outcomes of weaning fluid intake for the past 24 hours electrocardiogram (ECG) results

baseline arterial blood gas (ABG) levels Before weaning the client from mechanical ventilation, it's most important to have baseline ABG levels. During the weaning process, ABG levels will be checked to assess how the client is tolerating the procedure. Other assessment parameters are less critical. Measuring fluid volume intake and output is always important when a client is being mechanically ventilated. Prior attempts at weaning and ECG results are documented on the client's record, and the nurse can refer to them before the weaning process begins.

After a tonsillectomy, a client is being prepared for discharge. The nurse should instruct the client to report which sign or symptom immediately? throat pain bleeding difficulty swallowing difficulty talking

bleeding The nurse should instruct the client to report bleeding immediately. Delayed bleeding may occur when the healing membrane separates from the underlying tissue — usually 7 to 10 days postoperatively. Difficulty swallowing and throat pain are expected after a tonsillectomy and typically are present even before the client is discharged. Sudden difficulty talking wouldn't occur after discharge if the client could talk normally at the time of discharge, because swelling doesn't take that long to develop.

When evaluating the effectiveness of airway suctioning, the nurse should use which criterion? hollow sound on chest percussion decreased mucus production breath sounds clear on auscultation respirations unlabored

breath sounds clear on auscultation Auscultating for clear breath sounds is the most accurate way to evaluate the effectiveness of tracheobronchial suctioning. Auscultation should also be done to determine whether or not the client needs suctioning.Assessing for labored respirations is not as accurate in evaluating the effectiveness of tracheobronchial suctioning. A client may have labored breathing that is not affected by the presence or absence of tracheobronchial secretions.Percussion of the chest is useful for detecting masses or dense consolidation of lung tissue. It is not an accurate method for assessing the effectiveness of suctioning.Suctioning clears mucus but does not decrease its production.

Which task may be safely delegated to a licensed practical nurse (LPN)? changing the dressing of a client who underwent surgery 2 days ago administering an I.V. bolus dose of morphine sulfate to a client experiencing incisional pain teaching a client newly diagnosed with diabetes mellitus about insulin administration admitting a client who underwent a thoracotomy to the nursing unit from the postanesthesia care unit

changing the dressing of a client who underwent surgery 2 days ago The registered nurse may safely delegate dressing changes for the client who underwent surgery 2 days ago to the LPN. Teaching a client newly diagnosed with diabetes mellitus about insulin administration requires careful evaluation of the effectiveness of teaching and may not be delegated to an LPN. Admitting a client to the postanesthesia care unit is beyond the scope of practice for an LPN; LPNs aren't permitted to give I.V. push drugs.

A client is undergoing a complete physical examination as a requirement for college. When checking the client's respiratory status, the nurse observes respiratory excursion to help assess lung vibrations. chest movements. vocal sounds. breath sounds.

chest movements. The nurse observes respiratory excursion to help assess chest movements. Normally, thoracic expansion is symmetrical; unequal expansion may indicate pleural effusion, atelectasis, pulmonary embolus, or a rib or sternum fracture. The nurse assesses vocal sounds to evaluate air flow when checking for tactile fremitus; after asking the client to say the word "ninety-nine" the nurse palpates the vibrations transmitted from the bronchopulmonary system along the solid surfaces of the chest wall to the nurse's palms. The nurse assesses breath sounds during auscultation.

A nurse is caring for a client with deep vein thrombosis. Which change in assessment findings would the nurse be alert for related to the condition? bradypnea and bradycardia calf pain and redness hypertension and fever chest pain and dyspnea

chest pain and dyspnea The client with deep vein thrombosis is at risk for pulmonary embolism. As an embolus occludes a pulmonary artery, it blocks the supply of oxygenated blood to the heart, causing chest pain. It also blocks blood flow to the lungs, causing dyspnea. The client with pulmonary embolism may exhibit a cough that produces blood-tinged sputum. Hypertension, fever, bradypnea, and bradycardia aren't associated with pulmonary embolism. Calf pain and redness are associated with a deep vein thrombosis and would be expected.

A nurse has received a change-of-shift report on four clients. Which client should the nurse assess first? client with a recent lung transplant scheduled to begin pulmonary rehabilitation client with a pleural effusion who reports severe stabbing chest pain client experiencing tracheal deviation following a subclavian catheter insertion client with right-sided heart failure who has 4+ bilateral edema in the legs and feet

client experiencing tracheal deviation following a subclavian catheter insertion Tracheal deviation suggests possible tension pneumothorax, which is a medical emergency and needs to be evaluated immediately. Edema in a client with right-sided heart failure is a chronic condition and expected, it is not an emergency. Stabbing chest pain is expected with a pleural effusion and is also not an emergency situation. Pulmonary rehabilitation is completed by respiratory therapy and does not require the attention of the nurse.

A nurse is caring for a client who had a chest tube inserted 12 hours ago for treatment of a pleural effusion. Which assessment is most important in determining the client's response to the treatment? serous drainage in the collection chamber client resting quietly without reports of pain client verbalization of decreased dyspnea intermittent bubbling in the water seal chamber

client verbalization of decreased dyspnea Once some of the fluid has been removed via the chest tube drainage system, the client should feel relief and have a decrease in dyspnea. Serous drainage and bubbling in the water seal chamber are normal findings with the insertion of a chest tube, but do not indicate how the client is responding to the treatment.

A nurse is assigned to triage the care of four clients. Which client should the nurse assess first? client with a "scratchy throat" and a positive rapid strep antigen test client with throat cancer who has severe fatigue following radiation therapy client with a sore throat who now has a muffled voice and is drooling client with a history of a total laryngectomy whose stoma is red and inflamed

client with a sore throat who now has a muffled voice and is drooling The clinical manifestations of a muffled voice and drooling suggest a possible peritonsillar abscess that could lead to an airway obstruction. This requires rapid assessment and potential treatment. The remaining clients are stable and have expected symptoms that correspond to their medical diagnoses. Their treatment can wait until after the determination and treatment of the airway obstruction.

A client being treated for complications of chronic obstructive pulmonary disease needs to be intubated. The client has previously discussed their wish to not be intubated with the client's partner of 5 years, whom the client has designated as healthcare power of attorney. The client's children want their parent to be intubated. A nurse caring for this client knows that the client's partner is responsible for national legislation regarding surrogate decision makers. clients commonly confer healthcare power of attorney on someone who shares their personal values and beliefs. the children's biological relationship with their parent supersedes the partner's wishes. healthcare providers must honor the children's wishes to avoid a lawsuit.

clients commonly confer healthcare power of attorney on someone who shares their personal values and beliefs. The healthcare power of attorney is someone who can make decisions when the client can't. Clients tend to select individuals who share their personal values and beliefs as their healthcare power of attorney. Family members and designated surrogates don't always agree; state laws regarding surrogate decision makers may differ. The legal rights of a healthcare power of attorney in regards to healthcare decisions supersede those of family members. The law designates the healthcare power of attorney as the person to make decision; violating this designation could result in a lawsuit.

The nurse is assessing the respiratory status of a client who is experiencing an exacerbation of chronic obstructive pulmonary disease (COPD) secondary to an upper respiratory tract infection. Which finding is expected? coarse crackles and rhonchi normal chest movement prolonged inspiration normal breath sounds

coarse crackles and rhonchi Exacerbations of COPD are commonly caused by respiratory infections. Coarse crackles and rhonchi would be auscultated as air moves through airways obstructed with secretions. In COPD, breath sounds are diminished because of an enlarged anteroposterior diameter of the chest. Expiration, not inspiration, becomes prolonged. Chest movement is decreased as lungs become overdistended.

What is the best way for the nurse to position a chest tube for a client to prevent dislocation? coiled flat and secured in dependent loops along the side of the bed coiled flat on the bed and secured without putting tension on the tube coiled flat and secured to the bedrail coiled flat on the bed and positioned loosely

coiled flat on the bed and secured without putting tension on the tube Tubing that is coiled flat on the bed and secured without putting tension of the tube maintains a patent, free draining system. This prevents fluid accumulation and decreases the risk of infection, atelectasis, and tension pneumothorax. The other choices all have risks associated with becoming disconnected.

Which performance improvement strategy helps prevent adverse reactions to blood products? priming the blood administration tubing with normal saline solution obtaining baseline vital signs confirming client identification with two qualified health professionals instructing the client about the signs and symptoms of a blood reaction

confirming client identification with two qualified health professionals The client must be correctly identified to prevent a life-threatening adverse blood reaction. Obtaining vital signs, instructing the client about the signs and symptoms of a blood reaction, and priming the blood administration tubing with normal saline solution are key steps in the blood administration procedure; however, they don't prevent adverse reactions.

For a client with a sucking stab wound in the chest wall, the nurse should first: prepare for endotracheal intubation. cover the wound with a petroleum-impregnated dressing. administer oxygen. notify the health care provider.

cover the wound with a petroleum-impregnated dressing. The first course of action for a client with a sucking chest wound is to stop air from entering the chest cavity. Air entry will cause the lung to collapse. Stopping air entry is best done in an emergency situation by applying an air-occlusive dressing over the wound.The nurse can next notify the health care provider. Starting oxygen therapy and preparing for endotracheal intubation may be necessary later, but neither has the same priority on admission as closing the wound.

The nurse is caring for a client who is having an acute asthma attack. The nurse should notify the health care provider when the client has which symptom? persistent cough tenacious, thick sputum loud wheezing decreased breath sounds

decreased breath sounds Diminished breath sounds during an acute asthma attack are a serious sign of airway obstruction, fatigue, and impending respiratory failure. Wheezing, coughing, and the production of sputum indicate the presence of airflow through the lungs and are less ominous symptoms.

A client with lung cancer develops pleural effusion. During chest auscultation, which breath sound should the nurse expect to hear? decreased breath sounds wheezes rhonchi crackles

decreased breath sounds In pleural effusion, fluid accumulates in the pleural space, impairing transmission of normal breath sounds. Because of the acoustic mismatch, breath sounds are diminished. Crackles commonly accompany atelectasis, interstitial fibrosis, and left-sided heart failure. Rhonchi suggest secretions in the large airways. Wheezes result from narrowed airways, such as in asthma, chronic obstructive pulmonary disease, and bronchitis.

The nurse is performing a respiratory assessment on a client who has a pleural effusion. Which breath sound is expected for this client? hyperresonance on percussion decreased breath sounds on the affected side normal bronchial breath sounds wheezing on auscultation

decreased breath sounds on the affected side A pleural effusion is a collection of fluid between the pleural layers of the lung. The effusion decreases chest wall movement on the affected side. The nurse should expect the breath sounds to be decreased or diminished over the affected area. Because of the presence of fluid, percussion would elicit dullness, not hyperresonance. The nurse should not expect to hear wheezing on auscultation.

A home health nurse sees a client with end-stage chronic obstructive pulmonary disease (COPD). An outcome identified for this client is preventing infection. Which finding indicates that this outcome has been met? decreased oxygen requirements increased white blood cell count increased sputum production decreased activity tolerance

decreased oxygen requirements A client who is free from infection will most likely have decreased oxygen requirements. A client with infection will display increased sputum production, fever, shortness of breath, decreased activity tolerance, and increased oxygen requirements. Elevated white blood cell count may be indicative of infection.

The nurse administers two 325-mg aspirin every 4 hours to a client with pneumonia. The nurse should evaluate the outcome of administering the drug by assessing which findings? Select all that apply. decreased respiratory rate decreased pain when breathing prolonged clotting time decreased temperature increased ability to expectorate secretions

decreased pain when breathing decreased temperature Aspirin is administered to clients with pneumonia because it is an analgesic that helps control chest discomfort and an antipyretic that helps reduce fever. Aspirin has an anticoagulant effect, but that is not the reason for prescribing it for a client with pneumonia, and the use of the drug will be short term. Aspirin does not affect the respiratory rate and does not facilitate expectoration of secretions.

The nurse is auscultating the lung sounds of a client with long-standing emphysema. Which lung sounds are expected for this client? pleural friction rub diminished breath sounds fine crackles stridor

diminished breath sounds In emphysema, the anteroposterior diameter of the chest wall is increased. As a result, the client's breath sounds may be diminished. Fine crackles are present when there is fluid in the lungs. Stridor occurs as a result of a partially obstructed larynx or trachea; stridor can be heard without auscultation. A pleural friction rub is present when pleural surfaces are inflamed and rub together.

An adult with a history of chronic obstructive pulmonary disease (COPD) and metastatic carcinoma of the lung has not responded to radiation therapy and is being admitted to the hospice program. The nurse should conduct a focused assessment for which symptom? dyspnea peripheral edema pleural friction rub ascites

dyspnea Dyspnea is a distressing symptom in clients with advanced cancer including metastatic carcinoma of the lung, previous radiation therapy, and coexisting COPD. Ascites does occur in clients with metastatic carcinoma; however, in the client with COPD and lung cancer, dyspnea is a more common finding. A pleural friction rub is usually associated with pneumonia, pleurisy, or pulmonary infarct.

A nurse is assessing a client who comes to the clinic for care. Which findings in this client suggest bacterial pneumonia? dyspnea and wheezing hemoptysis and dysuria nonproductive cough and normal temperature sore throat and abdominal pain

dyspnea and wheezing In a client with bacterial pneumonia, retained secretions cause dyspnea, and respiratory tract inflammation causes wheezing. Bacterial pneumonia also produces a productive cough and fever, rather than a nonproductive cough and normal temperature. Sore throat occurs in pharyngitis, not bacterial pneumonia. Abdominal pain is characteristic of a GI disorder, unlike chest pain, which can reflect a respiratory infection such as pneumonia. Hemoptysis and dysuria aren't associated with pneumonia.

After undergoing a left thoracotomy, a client has a chest tube in place. When caring for this client, the nurse must encourage coughing and deep breathing. clamp the chest tube once every shift. report fluctuations in the water-seal chamber. milk the chest tube every 2 hours.

encourage coughing and deep breathing. When caring for a client who's recovering from a thoracotomy, the nurse should encourage coughing and deep breathing to prevent pneumonia. Fluctuations in the water-seal chamber are normal. Clamping the chest tube could cause a tension pneumothorax. Chest tube milking is controversial and should be done only to remove blood clots that obstruct the flow of drainage.

A nurse is caring for a client who has a tracheostomy and temperature of 103° F (39.4° C). Which intervention will most likely lower the client's arterial blood oxygen saturation? encouragement of coughing incentive spirometry use of a cooling blanket endotracheal suctioning

endotracheal suctioning Endotracheal suctioning removes secretions as well as gases from the airway and lowers the arterial oxygen saturation (SaO2) level. Coughing and using an incentive spirometer improve oxygenation and should raise or maintain oxygen saturation. Because of superficial vasoconstriction, using a cooling blanket can lower peripheral oxygen saturation readings, but SaO2 levels wouldn't be affected.

When teaching a client with chronic obstructive pulmonary disease to conserve energy, what instruction should the nurse give the client about breathing when lifting heavy objects? Lift the object by: taking a deep breath and holding it. exhaling but before inhaling. inhaling through an open mouth. exhaling through pursed lips.

exhaling through pursed lips. Exhaling requires less energy than inhaling. Therefore, lifting while exhaling saves energy and reduces perceived dyspnea. Pursing the lips prolongs exhalation and provides the client with more control over breathing. Lifting after exhaling but before inhaling is similar to lifting with the breath held. This should not be recommended because it is similar to the Valsalva maneuver, which can stimulate cardiac arrhythmias.

A client on mechanical ventilation is receiving pancuronium I.V. as needed. Which assessment finding indicates that the client needs another pancuronium dose? fighting the ventilator finger movement lip movement leg movement

fighting the ventilator Pancuronium, a nondepolarizing blocking agent, is used for muscle relaxation and paralysis. It assists mechanical ventilation by promoting endotracheal intubation and paralyzing the client so the client breathes in synchrony with the ventilator. Fighting the ventilator is a sign that the client needs another pancuronium dose. The nurse should administer a dose I.V. every 20 to 60 minutes. Movement of the legs, fingers, or lips has no effect on the ventilator and therefore isn't used to determine the need for another dose.

The nurse is assessing a client with chronic obstructive pulmonary disease. Which finding requires immediate intervention? inability to speak distant heart sounds diminished lung sounds pursed lip breathing

inability to speak Inability to speak could indicate respiratory distress. Pulsed lip breathing, while it is an abnormal finding is not indicative of respiratory distress. Distant heart sounds could indicate heart failure but are not indicative of any distress.

A client with chronic sinusitis comes to the outpatient department complaining of headache, malaise, and a nonproductive cough. When examining the client's paranasal sinuses, the nurse detects tenderness. To evaluate this finding further, the nurse should transilluminate the frontal sinuses only. frontal and maxillary sinuses. sphenoidal and ethmoidal sinuses. sphenoidal sinuses only.

frontal and maxillary sinuses. After detecting tenderness of the paranasal sinuses, the nurse should transilluminate both the frontal and maxillary sinuses; lack of illumination may indicate sinus congestion and pus accumulation. The sphenoidal and ethmoidal sinuses can't be transilluminated because of their location.

A positive tuberculin skin test indicates that a client will develop full-blown tuberculosis. has produced an immune response. is actively immune to tuberculosis. has an active case of tuberculosis.

has produced an immune response. The tuberculin skin test is based on the antigen/antibody response and will show a positive reaction after an individual has been exposed to tuberculosis and has formed antibodies to the tuberculosis bacteria. Thus, a positive tuberculin skin test indicates the production of an immune response. Exposure doesn't confer immunity. A positive test doesn't confirm that a person has (or will develop) tuberculosis.

A client is receiving streptomycin to treat tuberculosis. What should the nurse evaluate to determine an adverse effect of the drug? hearing loss IV infiltration difficulty swallowing decreased serum creatinine

hearing loss Streptomycin can cause toxicity to the eighth cranial nerve, which is responsible for hearing, balance, and body position sense. Nephrotoxicity is a side effect that would be indicated with an increase in creatinine. Streptomycin does not cause difficulty in swallowing. Streptomycin is given via intramuscular injection.

A client who has been recently extubated has shortness of breath. The nurse reports the client's discomfort and the results of the recently prescribed arterial blood gas analysis to the health care provider (HCP). After reviewing the report of the complete blood count (see report), the nurse should also report which results to the HCP? HA1c hemoglobin and hematocrit monocytes prothrombin time (PT)

hemoglobin and hematocrit The nurse should review the CBC with differential to evaluate the client's hemoglobin and hematocrit, which are abnormal and should be reported to the HCP. Anemia leads to decreased oxygen-carrying capacity of the blood. A client unable to compensate for the anemia may experience a profound sense of dyspnea. There has been a significant drop in the hemoglobin and hematocrit since the previous report, and these should be reported to the HCP. The monocytes are within normal range. HA1c is a laboratory test evaluating glycosylated hemoglobin and is in the normal range. This test is used to diagnose diabetes and/or monitor diabetic glucose control over time. PT is a coagulation study reflecting liver function and clotting time and is in the normal range.

A client diagnosed with tuberculosis is taking the prescribed chemotherapy of isoniazid, rifampin, and pyrazinamide. Although side effects are rare, the nurse should assess the client for which side effect of this drug combination? nephrotoxicity hepatotoxicity optic neuritis ototoxicity

hepatotoxicity The major side effect of these three drugs is liver toxicity. While the client is taking these drugs, the nurse should carefully monitor the client's liver function tests.Ototoxicity and nephrotoxicity are side effects of other drugs used to treat TB, such as streptomycin, kanamycin, and capreomycin.Optic neuritis can be a rare side effect of isoniazid.

The nurse is aware that the best position for a client with impaired gas exchange is what? semi-Fowler's side-lying high Fowler's Sims

high Fowler's For a client with impaired gas exchange, high Fowler's position is the best position because it allows maximal chest expansion. If the client cannot tolerate high Fowler's position, semi-Fowler's is the next best choice, because it increases comfort and allows for chest expansion. The lateral decubitus and supine positions do not promote chest expansion. Sims position is recommended for perineal inspection.

Which diet would be most appropriate for a client with chronic obstructive pulmonary disease (COPD)? low-fat, low-cholesterol diet low-sodium diet high-calorie, high-protein diet bland, soft diet

high-calorie, high-protein diet The client should eat high-calorie, high-protein meals to maintain nutritional status and prevent weight loss that results from the increased work of breathing. The client should be encouraged to eat small, frequent meals. A low-fat, low-cholesterol diet is indicated for clients with coronary artery disease. The client with COPD does not necessarily need to follow a sodium-restricted diet, unless otherwise medically indicated. There is no need for the client to eat bland, soft foods.

A nurse consulting with a nutrition specialist knows it's important to consider a special diet for a client with chronic obstructive pulmonary disease (COPD). Which diet is appropriate for this client? full-liquid 1,800-calorie ADA low-fat high-protein

high-protein Breathing is more difficult for clients with COPD, and increased metabolic demand puts them at risk for nutritional deficiencies. These clients must have a high intake of protein for increased calorie consumption. Full liquids, 1,800-calorie ADA, and low-fat diets aren't appropriate for a client with COPD.

On arrival at the intensive care unit, a critically ill client suffers respiratory arrest and is placed on mechanical ventilation. The physician orders pulse oximetry to monitor the client's arterial oxygen saturation (SaO2) noninvasively. Which vital sign abnormality may alter pulse oximetry values? tachypnea hypotension fever tachycardia

hypotension Hypotension, hypothermia, and vasoconstriction may alter pulse oximetry values by reducing arterial blood flow. Likewise, movement of the finger to which the oximeter is applied may interfere with interpretation of SaO2. All of these conditions limit the usefulness of pulse oximetry. Fever, tachypnea, and tachycardia don't affect pulse oximetry values directly.

A client had posterior packing inserted to control a severe nosebleed. After insertion of the packing, the nurse should observe the client for which finding? hypoventilation vertigo Bell's palsy loss of gag reflex

hypoventilation Posterior packing may alter the respiratory status of the client, especially in older adults clients, causing hypoventilation. Clients should be observed carefully for changes in level of consciousness, respiratory rate, and heart rate and rhythm after the insertion of the packing. Vertigo does not occur as a result of the insertion of posterior packing. Bell's palsy, a disorder of the seventh cranial nerve, is not associated with epistaxis or nasal packing. Loss of gag reflex does not occur as a result of the insertion of posterior packing.

A client is admitted to the hospital with a diagnosis of a pulmonary embolism. Which problem should the nurse address first? nonproductive cough activity intolerance impaired gas exchange difficulty breathing

impaired gas exchange Emboli obstruct blood flow, leading to a decreased perfusion of the lung tissue. Because of the decreased perfusion, a ventilation-perfusion mismatch occurs, causing hypoxemia to develop. Arterial blood gas analysis typically will indicate hypoxemia and hypocapnia. A priority objective in the treatment of pulmonary emboli is maintaining adequate oxygenation. A productive cough and activity intolerance do not indicate impaired gas exchange. The client does not demonstrate an ineffective breathing pattern; rather, the problem of impaired gas exchange is caused by the inability of blood to flow through the lung tissue.

A client with asthma has been taking theophylline as ordered. Now, the client's blood theophylline level is 4.8 mcg/ml. Which dosage change will the physician order? decrease the dose. increase the dose. maintain the dose. omit a dose.

increase the dose. When the client's serum theophylline concentration falls below the therapeutic level, the daily dosage of the ordered methylxanthine agent should be increased by up to 25%, and the serum theophylline concentration should be rechecked before further dosage changes are made. Decreasing the dose, maintaining the dose, or omitting a dose wouldn't help increase the serum theophylline level to therapeutic levels.

When assessing a client with advanced chronic obstructive pulmonary disease (COPD) which are expected findings? underdeveloped neck muscles collapsed neck veins increased chest excursions with respiration increased anteroposterior chest diameter

increased anteroposterior chest diameter Increased anteroposterior chest diameter is characteristic of advanced COPD. Air is trapped in the overextended alveoli, and the ribs are fixed in an inspiratory position. The result is the typical barrel-chested appearance. Overly developed, not underdeveloped, neck muscles are associated with COPD because of their increased use in the work of breathing. Distended, not collapsed, neck veins are associated with COPD as a symptom of the heart failure that the client may experience secondary to the increased workload on the heart to pump blood into the pulmonary vasculature. Diminished, not increased, chest excursion is associated with COPD.

A client is critically ill with sepsis. The nurse expects what assessment finding related to compensatory mechanisms attempting to maintain normal pH? increased body temperature increased urine output decreased blood pressure increased respiratory rate

increased respiratory rate The critically ill client with sepsis is at risk for decreased perfusion of tissues and organs, which leads to lactic acid production. This causes the client to experience metabolic acidosis. To correct the acidosis, the lungs eliminate carbonic acid by blowing off more CO2 via an increased respiratory rate. It is the respiratory system that compensates for metabolic acidosis, not the renal system. Blood pressure will be low in the client with sepsis, but blood pressure is not a compensatory mechanism for pH imbalances. While body temperature can affect acid base balance, this is not how the body compensates for metabolic acidosis.

A physician orders a palliative care consult for a client with end-stage chronic obstructive pulmonary disease who wishes no further medical intervention. Which step should the nurse anticipate based on the nurse's knowledge of palliative care? decreasing administration of pain medications reducing oxygen requirements increasing the need for antianxiety agents decreasing the use of bronchodilators

increasing the need for antianxiety agents The nurse should anticipate that the physician will increase antianxiety agents during treatment to maintain comfort throughout the dying process. Bronchodilators, pain medications, and home oxygen therapy help promote client comfort. Therefore, they should be continued as part of palliative care.

A client who sustained a pulmonary contusion in a motor vehicle crash develops a pulmonary embolism. Which nursing diagnosis takes priority with this client? impaired circulation related to blood clot risk for vascular trauma related to pulmonary emboli acute pain related to tissue trauma ineffective breathing pattern related to tissue trauma

ineffective breathing pattern related to tissue trauma Although all of these nursing diagnoses are appropriate for this client, ineffective breathing pattern takes priority. According to Maslow's hierarchy of needs, air is essential to maintain life and is assigned the highest priority, along with the other physiologic needs, such as food, elimination, temperature control, sex, movement, rest, and comfort.

A client is reporting shortness of breath. The nurse finds the client's assessment includes dyspnea, diaphoresis, and slight confusion. Concerned that the client may have an air embolism, how should the nurse position the client? right side in the deep Trendelenburg position left side in semi-Fowler's position in the semi-Fowler's position left side in the deep Trendelenburg position

left side in the deep Trendelenburg position Positioning the client on the left side in the deep Trendelenburg position keeps the air in the right atrium and out of pulmonary circulation, preventing obstruction of the right ventricle. The other choices are not correct based on movement of an air embolism and care to stop it from moving further.

After the nurse informs the surgeon that a chest tube is malfunctioning, the health care provider asks the nurse to reposition the tube and obtain a chest radiograph. The nurse should: follow the prescription as requested by the surgeon. report the surgeon to the Ethics Committee. report the surgeon to the nursing supervisor. inform the surgeon this is not within the safe scope of practice.

inform the surgeon this is not within the safe scope of practice. Initially, the nurse needs to inform the surgeon that the task is outside the scope of nursing practice.If the surgeon still requests the activity, the nurse should refuse to perform the task and should follow the chain of communication for reporting unsafe practice according to the facility's policy.The nurse must not comply with any prescription that goes beyond the scope of nursing practice.

The nurse is caring for a client of African descent who is having increased respiratory difficulty. When assessing the client's oxygen status, which assessment technique is used to evaluate client cyanosis? Select all that apply. inspecting the nailbeds assessing the earlobe assessing the lips and gums noting patchy skin color across the abdomen inspecting the conjunctiva

inspecting the nailbeds inspecting the conjunctiva assessing the lips and gums The color of a client's skin is determined by the amount of pigment it contains and the blood flowing through it. In clients with highly pigmented skin, cyanosis is accurately detected by inspecting the conjunctiva and nailbeds and assessing the oral mucous membranes including the lips and gums. The other options do not provide the assessment for cyanosis.

A client requires long-term ventilator therapy. The client has a tracheostomy in place and requires frequent suctioning. Which technique should the nurse use? intermittent suction while advancing the catheter continuous suction while withdrawing the catheter intermittent suction while withdrawing the catheter continuous suction while advancing the catheter

intermittent suction while withdrawing the catheter To prevent hypoxia, the nurse should use intermittent (not continuous) suction while withdrawing the catheter. Suctioning shouldn't last more than 10 seconds at a time. Neither intermittent nor continuous suctioning should be applied while the catheter is being advanced.

A client is prescribed metaproterenol via a metered-dose inhaler, two puffs every 4 hours. The nurse instructs the client to report which adverse effect? irregular heartbeat constipation pedal edema decreased pulse rate

irregular heartbeat Irregular heartbeats should be reported promptly to the care provider. Metaproterenol may cause irregular heartbeat, tachycardia, or anginal pain because of its adrenergic effect on beta-adrenergic receptors in the heart. It is not recommended for use in clients with known cardiac disorders. Metaproterenol does not cause constipation, pedal edema, or bradycardia.

A client had a Mantoux test result of an 8-mm induration. When should the nurse interpret the test as positive? When the client: has no known risk factors. is immunocompromised. works as a health care provider in a hospital. lives in a long-term care facility.

is immunocompromised. An induration (palpable raised hardened area of skin) of more than 5 to 15 mm (depending upon the person's risk factors) to 10 Mantoux units is considered a positive result, indicating TB infection. An induration of greater than 5 mm is found in HIV-positive individuals, those with recent contacts with persons with TB, persons with nodular or fibrotic changes on chest x-ray consistent with old healed TB, or clients with organ transplants or immunosuppressed. An induration of greater than 10 mm is positive, and the client may be a recent arrival (less than 5 years) from high-prevalent countries, injection drug user, resident or an employee of high-risk congregate settings (e.g., prisons, long-term care facilities, hospitals, homeless shelters), or mycobacteriology lab personnel. Persons with clinical conditions that place them at high risk (e.g., diabetes, prolonged corticosteroid therapy, leukemia, end-stage renal disease, chronic malabsorption syndromes, low body weight), a child less than 4 years of age, or a child or adolescents exposed to adults in high-risk categories.

A nurse is completing annual cardiopulmonary resuscitation training. The class instructor states that a client has fallen off a ladder and is lying on the back. The client is unconscious and not breathing. What technique should the nurse use to open the client's airway? head tilt-chin lift abdominal thrust log roll to the side jaw-thrust

jaw-thrust If a neck or spine injury is suspected, the jaw-thrust maneuver should be used to open the client's airway. To perform this maneuver, the nurse should be positioned at the client's head and rest the thumbs on the client's lower jaw, near the corners of the mouth. The nurse should then grasp the angles of the lower jaw with the fingers and lift the jaw forward. The head tilt-chin lift maneuver is used to open the airway when a neck or spine injury is not suspected. To perform this maneuver, the nurse places two fingers on the chin and lifts while pushing down on the forehead with the other hand. The abdominal thrust is used to relieve severe or complete airway obstruction caused by a foreign body. Log rolling the client will not open the airway.

After a bronchoscopy with biopsy, the nurse assesses the client. The nurse should report which finding to the health care provider? green sputum dry cough hemoptysis laryngeal stridor

laryngeal stridor Laryngeal stridor is characteristic of respiratory distress from inflammation and swelling after bronchoscopy. It must be reported immediately. Green sputum indicates infection and would occur 3 to 5 days after bronchoscopy. A mild cough or hemoptysis is typical after bronchoscopy. If a tissue biopsy specimen was obtained, sputum may be blood-streaked for several days.

The nurse administers theophylline to a client. When evaluating the effectiveness of this medication, what is an expected outcome? suppression of the client's respiratory infection. thinning of tenacious, purulent sputum. less difficulty breathing decrease in bronchial secretions.

less difficulty breathing Theophylline is a bronchodilator that is administered to relax airways and decrease dyspnea. Theophylline is not used to treat infections and does not decrease or thin secretions.

Which finding in a client diagnosed with asthma would require a nurse to take immediate action? lethargy cough diaphoresis anhidrosis

lethargy Lethargy can be a manifestation of status asthmaticus. Anhidrosis, cough, and diaphoresis should be further assessed.

The nurse is caring for a child with history of strep throat. Upon current assessment, the child reports abdominal pain and joint achiness. Which laboratory data would the nurse communicate to the health care provider immediately? normal erythrocyte sedimentation rate anemia leukocytosis low hemoglobin level

leukocytosis Leukocytosis can be seen as an immune response triggered by colonization of the pharynx with group A streptococci. This finding is expected in a client with rheumatic fever. Laboratory data indicating anemia or a low hemoglobin level will need to be addressed but are not critical and associated with the current disease process. A marginal erythrocyte sedimentation rate would be communicated on the laboratory report.

After teaching a client how to instill nose drops, the nurse evaluates that the client's technique is correct when the client: uses sterile technique when handling the dropper. blows the nose gently after instilling the medicine. lies supine for several minutes after instilling the drops. uses a new dropper for each medication instillation.

lies supine for several minutes after instilling the drops. The client should be instructed to lie supine with the head tilted back for several minutes after instillation of the nose drops to ensure adequate absorption of the medication by the nasal mucosa.Because the nose is not sterile, sterile technique is not necessary.The client should blow the nose gently before, not after, administering the medication.The dropper should be cleaned after each use; a new dropper is not necessary for each instillation.

A client hospitalized for treatment of a pulmonary embolism develops respiratory alkalosis. Which clinical findings commonly accompany respiratory alkalosis? abdominal pain or diarrhea hallucinations or tinnitus light-headedness or paresthesia nausea or vomiting

light-headedness or paresthesia The client with respiratory alkalosis may complain of light-headedness or paresthesia (numbness and tingling in the arms and legs). Nausea, vomiting, abdominal pain, and diarrhea may accompany respiratory acidosis. Hallucinations and tinnitus rarely are associated with respiratory alkalosis or any other acid-base imbalance.

A client reports difficulty breathing and a sharp pain in the right side of the chest. The respiratory rate measures 40 breaths/minute. The nurse should assign highest priority to which care goal? relieving pain maintaining effective respirations maintaining an adequate circulatory volume reducing anxiety

maintaining effective respirations As suggested by the ABCs of cardiopulmonary resuscitation — airway, breathing, and circulation — the most important goal is to maintain a patent airway and effective respirations, regardless of the client's diagnosis or clinical presentation. Although maintaining an adequate circulatory volume, reducing anxiety, and relieving pain are pertinent for this client, they're secondary to maintaining effective respirations.

A client with chronic obstructive pulmonary disease (COPD) is intubated and placed on continuous mechanical ventilation. Which equipment is most important for the nurse to keep at this client's bedside? water-seal chest drainage set-up manual resuscitation bag oxygen analyzer tracheostomy cleaning kit

manual resuscitation bag The client with COPD depends on mechanical ventilation for adequate tissue oxygenation. The nurse must keep a manual resuscitation bag at the bedside to ventilate and oxygenate the client in case the mechanical ventilator malfunctions. Because the client doesn't have chest tubes or a tracheostomy, keeping a water-seal chest drainage set-up or a tracheostomy cleaning kit at the bedside isn't necessary. Although the nurse may keep an oxygen analyzer (pulse oximeter) on hand to evaluate the effectiveness of ventilation, this equipment is less important than the manual resuscitation bag.

A client with cystic fibrosis develops pneumonia. To decrease the viscosity of respiratory secretions, the physician orders acetylcysteine. Before administering the first dose, the nurse checks the client's history for asthma. Acetylcysteine must be used cautiously in a client with asthma because it is a respiratory depressant. is a respiratory stimulant. inhibits the cough reflex. may induce bronchospasm.

may induce bronchospasm. Acetylcysteine must be used cautiously in a client with asthma because it may induce bronchospasm. The drug isn't a respiratory depressant or stimulant. It's a mucolytic agent that decreases the viscosity of respiratory secretions by altering the molecular composition of mucus. Acetylcysteine doesn't inhibit the cough reflex.

A client in the emergency department is diagnosed with a communicable disease. When complications of the disease are discovered, the client is admitted to the hospital and placed in respiratory isolation. Which infection warrants airborne isolation? cholera mumps impetigo measles

measles Measles warrants airborne isolation, which aims to prevent transmission of disease by airborne nuclei droplets. Other infections necessitating respiratory isolation include varicella and tuberculosis. The mumps call for droplet isolation; impetigo, contact isolation; and cholera, enteric isolation.

A client has the following arterial blood gas (ABG) values: pH, 7.12; partial pressure of arterial carbon dioxide (PaCO2), 40 mm Hg; and bicarbonate (HCO3-), 15 mEq/L. These ABG values suggest which disorder? metabolic alkalosis respiratory acidosis metabolic acidosis respiratory alkalosis

metabolic acidosis This client's pH value is below normal, indicating acidosis. The HCO3- value also is below normal, reflecting an overwhelming accumulation of acids or excessive loss of base, which suggests metabolic acidosis. The PaCO2 value is normal, indicating absence of respiratory compensation. These ABG values eliminate respiratory alkalosis, respiratory acidosis, and metabolic alkalosis.

The nurse is assessing a client with a darker-skin tone in need of emergency care for acute respiratory distress. Which area would the nurse inspect when assessing for cyanosis in this client? earlobes mucous membranes nail beds lips

mucous membranes Skin color does not affect the mucous membranes. Therefore, the nurse can assess for cyanosis by inspecting the client's mucous membranes. The lips, nail beds, and earlobes are less-reliable indicators of cyanosis because they are affected by skin color.

A physician orders albuterol for a client with newly diagnosed asthma. When teaching the client about this drug, the nurse should explain that it may cause nasal congestion. nervousness. hyperkalemia. lethargy.

nervousness. Albuterol may cause nervousness. The inhaled form of the drug may cause dryness and irritation of the nose and throat, not nasal congestion; insomnia, not lethargy; and hypokalemia (with high doses), not hyperkalemia. Other adverse effects of albuterol include tremor, dizziness, headache, tachycardia, palpitations, hypertension, heartburn, nausea, vomiting, and muscle cramps.

A client admitted with a deep vein thrombosis abruptly sits up in bed, reports having difficulty breathing and has an arterial oxygen saturation of 88%. Which mode of oxygen delivery is most likely to improve these manifestations? nasal cannula face tent simple mask nonrebreather mask

nonrebreather mask A nonrebreather mask can deliver levels of the fraction of inspired oxygen (FIO2) as high as 100%. Other modes — simple mask, face tent, and nasal cannula — deliver lower levels of FIO2.

A client with asthma has been prescribed beclomethasone via metered-dose inhaler. To determine if the client has been rinsing the mouth after each use of the inhaler, the nurse should inspect the client's mouth for which health problem? ulceration gingival hyperplasia dental caries oral candidiasis

oral candidiasis Beclomethasone is an inhaled steroid used for the maintenance treatment of asthma. The steroid can precipitate overgrowth of fungus, such as oral Candida albicans. Rinsing the mouth well after each use decreases the incidence of oral fungal infections. Beclomethasone does not cause gingival hyperplasia, ulceration, or caries.

The client is taking triamcinolone acetonide inhalant to treat bronchial asthma. The nurse should assess the client for: hyperglycemia. gastric ulcer. oral candidiasis. fluid retention.

oral candidiasis. Acetonide inhalant is a corticosteroid. Use of a steroid inhaler can cause the client to develop oral candidiasis (thrush). It is important that the client rinse his or her mouth after using the inhaler.Acetonide inhalant does not lead to the development of systemic complications such as hyperglycemia, ulcers, or fluid retention.

A healthcare provider has entered orders for a client with chronic obstructive pulmonary disease (COPD). Which order should the nurse question? albuterol nebulizer treatments every 4 hours as needed oxygen increased to 3 L/minute if oxygen saturation is less than 94% on room air oxygen via nasal canula at 2 L/minute head of bed elevated 30-45°

oxygen increased to 3 L/minute if oxygen saturation is less than 94% on room air People with COPD retain CO2, which is the normal trigger for respiratory rate. In clients with COPD and high levels of CO2, oxygen levels trigger breathing. Too much oxygen and the body slows breathing. Clients with COPD may quit breathing completely when given oxygen at very high levels (greater than 2 L).

A nurse is evaluating a client with primary pulmonary hypertension for a heart-lung transplant. Which medication treatment would the nurse anticipate to be included in the plan of care? Select all that apply. aminoglycosides antihistamines oxygen therapy diuretics vasodilators sulfonamides

oxygen therapy diuretics vasodilators Pulmonary hypertension is an increase in blood pressure in the pulmonary arteries, pulmonary vein, or pulmonary capillary. This increase in pressure makes it more difficult for the blood to flow through the lungs. Oxygen, diuretics, and vasodilators are among the common medication therapies used to treat pulmonary hypertension. Other treatments include fluid restriction, digoxin, calcium channel blockers, β-adrenergic blockers, and bronchodilators. Aminoglycosides and sulfonamides are antibiotics used to treat infections. Antihistamines are indicated to treat allergies, pruritus, vertigo, nausea, and vomiting; to promote sedation; and to suppress cough.

Arterial blood gas analysis would reveal which finding related to acute respiratory failure? PaO2 80 mm Hg pH 7.35 PaCO2 32 mm Hg pH 7.24

pH 7.24 Acute respiratory failure (ARF) is defined as a decrease in the arterial oxygen tension (PaO2) to less than 50 mm Hg (hypoxemia) and an increase in arterial carbon dioxide tension (PaCO2) to greater than 50 mm Hg (hypercapnia), with a decreased arterial pH.

The nurse is interpreting blood gases for a client with acute respiratory distress syndrome (ARDS). Which set of arterial blood gas values does the nurse expect for this client? pH 7.29 PaCO2 36, HCO3 19 pH 7.35, PaCO2 46, HCO3 30 pH 7.25, PaCO2 48, HCO3 24 pH 7.30, PaCO2 28, HCO3 16

pH 7.25, PaCO2 48, HCO3 24 The nurse would expect a client with ARDS to exhibit respiratory acidosis. The results of pH 7.25, PaCO2 48, HCO3 24 indicate respiratory acidosis. Results of pH 7.29, PaCO2 36, HCO3 19 indicate metabolic acidosis and results of pH 7.30, PaCO2 28, HCO3 16 indicate metabolic acidosis with partial compensation, which would be expected in a client with a metabolic problem such as diabetic ketoacidosis. Results of pH 7.35, PaCO2 46, HCO3 30 indicate fully compensated respiratory acidosis, which would be expected for client with a chronic respiratory problem.

A nurse assesses arterial blood gas results for a client in acute respiratory failure (ARF). Which result is consistent with this disorder? pH 7.28, PaO2 50 mm Hg pH 7.36, PaCO2 32 mm Hg pH 7.46, PaO2 80 mm Hg pH 7.35, PaCO2 48 mm Hg

pH 7.28, PaO2 50 mm Hg ARF is defined as a decrease in the arterial oxygen tension (PaO2) to less than 50 mm Hg (hypoxemia) and an increase in arterial carbon dioxide tension (PaCO2) to greater than 50 mm Hg (hypercapnia), with an arterial pH of less than 7.35.

A nurse is reviewing arterial blood gas (ABG) results on four clients. Which results would the nurse report immediately to the healthcare provider? pH 7.36, PaO2 86 mmHg, PaCO2 55 mmHg, HCO3 30 mEq/L, and O2 sat 90% pH 7.35, PaO2 82 mmHg, PaCO2 44 mmHg, HCO3 22 mEq/L, and O2 sat 95% pH 7.31, PaO2 84 mmHg, PaCO2 50 mmHg, HCO3 24 mEq/L, and O2 sat 90% pH 7.45, PaO2 90 mmHg, PaCO2 28 mmHg, HCO3 32 mEq/L, and O2 sat 98%

pH 7.31, PaO2 84 mmHg, PaCO2 50 mmHg, HCO3 24 mEq/L, and O2 sat 90% This ABG of pH 7.31, PaO2 84 mmHg, PaCO2 50 mmHg, HCO3 24 mEq/L, and O2 sat 90% indicates uncompensated respiratory acidosis and would be reported to the healthcare provider immediately. The results pH 7.35, PaO2 82 mmHg, PaCO2 44 mmHg, HCO3 22 mEq/L, and O2 sat 95% are normal values. The results pH 7.45, PaO2 90 mmHg, PaCO2 28 mmHg, HCO3 32 mEq/L, and O2 sat 98% and pH 7.36, PaO2 86 mmHg, PaCO2 55 mmHg, HCO3 30 mEq/L, and O2 sat 90% both indicate fully compensated acid-base imbalances; they would not be the priority for reporting to the healthcare provider.

The nurse has received lab reports for several clients undergoing care. Which set of arterial blood gas (ABG) results will the nurse investigate first? pH 7.34, partial pressure of arterial carbon dioxide (PaCO2) 36 mmHg, partial pressure of arterial oxygen (PaO2) 95 mmHg, bicarbonate (HCO3-) 20 mEq/L pH 7.35, PaCO2 48 mmHg, PaO2 91 mmHg, and HCO3- 28 mEq/L pH 7.49, PaCO2 30 mmHg, PaO2 75 mmHg, and HCO3- 22 mEq/L pH 7.47, PaCO2 43 mmHg, PaO2 99 mmHg, and HCO3- 29 mEq/L

pH 7.49, PaCO2 30 mmHg, PaO2 75 mmHg, and HCO3- 22 mEq/L The ABG results pH 7.49, PaCO2 30 mmHg, PaO2 75 mmHg, and HCO3- 22 mEq/L indicate respiratory alkalosis. The pH level is increased, the PaCO2 levels are decreased and the HCO3 is normal. The decreased PaO2 indicates this client is in respiratory distress. Therefore, the nurse would investigate this result first. Normal values are pH 7.35 to 7.45, PaCO2 35 to 45 mmHg, and HCO3- 22 to 26 mEq/L. Results of pH 7.35, PaCO2 48 mmHg, PaO2 91 mmHg, and HCO3- 28 mEq/L indicate a fully compensated respiratory acidosis, making this less urgent. Results of pH 7.47, PaCO2 43 mmHg, PaO2 99 mmHg, and HCO3- 29 mEq/L indicate metabolic alkalosis, which is not the priority. Results of pH 7.34, PaCO2 36 mmHg, PaO2 95 mmHg, HCO3- 20 mEq/L indicate mild metabolic acidosis, which would also be less urgent than the respiratory alkalosis.

The nurse is analyzing the arterial blood gas (AGB) results of a client diagnosed with severe pneumonia. What ABG results are most consistent with this diagnosis? pH: 7.20, PaCO2: 65 mm Hg, HCO3-: 26 mEq/L pH: 7.32, PaCO2: 40 mm Hg, HCO3-: 18 mEq/L pH: 7.50, PaCO2: 30 mm Hg, HCO3-: 24 mEq/L pH: 7.42, PaCO2: 45 mm Hg, HCO3-: 22 mEq /L

pH: 7.20, PaCO2: 65 mm Hg, HCO3-: 26 mEq/L Respiratory acidosis is a clinical disorder in which the pH is less than 7.35 and the PaCO2 is greater than 42 mm Hg and a compensatory increase in the plasma HCO3- occurs. It may be either acute or chronic. The ABG of pH: 7.32, PaCO2: 40 mm Hg, HCO3-: 18 mEq/L indicates metabolic acidosis. The ABGs of pH: 7.50, PaCO2: 30 mm Hg, and HCO3-: 24 mEq/L indicate respiratory alkalosis. The ABGs of pH 7.42, PaCO2: 45 mm Hg, and HCO3-: 22 mEq/L indicate a normal result/no imbalance.

A client is receiving supplemental oxygen. When determining the effectiveness of oxygen therapy, which arterial blood gas value is most important? pH partial pressure of arterial oxygen (PaO2) bicarbonate (HCO3-) partial pressure of arterial carbon dioxide (PaCO2)

partial pressure of arterial oxygen (PaO2) The most significant and direct indicator of the effectiveness of oxygen therapy is the PaO2 value. Based on the PaO2 value, the nurse may adjust the type of oxygen delivery (cannula, Venturi mask, or mechanical ventilator), flow rate, and oxygen percentage. The other options reflect the client's ventilation status, not oxygenation. The pH, HCO3-, and PaCO2

The nurse is caring for a client with acute respiratory distress syndrome. What portion of arterial blood gas results does the nurse find most concerning, requiring intervention? partial pressure of arterial oxygen (PaO2) of 69 mm Hg pH of 7.29 bicarbonate (HCO3-) of 28 mEq/L partial pressure of arterial carbon dioxide (PaCO2) of 51 mm Hg

partial pressure of arterial oxygen (PaO2) of 69 mm Hg In acute respiratory failure, administering supplemental oxygen elevates the PaO2. In acute respiratory distress syndrome, elevation of the PaO2 requires positive end-expiratory pressure. In both situations, the PaCO2 is elevated and the pH and HCO3- are depressed.

The nurse is teaching a client with chronic obstructive pulmonary disease (COPD) to assess for signs and symptoms of right-sided heart failure. Which sign or symptom should be included in the teaching plan? clubbing of nail beds hypertension increased appetite peripheral edema

peripheral edema Right-sided heart failure is a complication of COPD that occurs because of pulmonary hypertension. Signs and symptoms of right-sided heart failure include peripheral edema, jugular venous distention, hepatomegaly, and weight gain due to increased fluid volume. Clubbing of nail beds is associated with conditions of chronic hypoxemia. Hypertension is associated with left-sided heart failure. Clients with heart failure have decreased appetites.

Clients who have had active tuberculosis are at risk for recurrence. Which condition increases that risk? physical and emotional stress rest and inactivity cool and damp weather active exercise and exertion

physical and emotional stress Tuberculosis can be controlled but never completely eradicated from the body. Periods of intense physical or emotional stress increase the likelihood of recurrence. Clients should be taught to recognize the signs and symptoms of a potential recurrence. Weather and activity levels are not related to recurrences of tuberculosis.

The nurse is aware that frequent repositioning in bed will assist in the prevention of which condition for a client? pneumonia arterial thrombosis intravenous infiltration postural hypotension

pneumonia By frequently changing positions in bed, the client can prevent the development of pneumonia, urinary stasis, and deep vein thrombosis. These movements promote blood, oxygen, and fluid circulation throughout the body systems and prevent stasis. Postural hypotension can often be associated with medications and no information is given about this in the question. Arterial thrombosis is incorrect because decreased movement would more likely result in a venous thrombosis.

Which vaccine should a nurse encourage a client with chronic obstructive pulmonary disease (COPD) to receive? Select all that apply. hepatitis B pneumonia pertussis influenza varicella

pneumonia pertussis influenza Clients with COPD are more susceptible to respiratory infections, so they should be encouraged to receive the influenza, pneumococcal, and pertussis vaccines. Clients with COPD are not at high risk for varicella or hepatitis B.

A client is brought to the emergency department following an automobile accident. Physical assessment reveals tachycardia, dyspnea, and absent breath sounds over the right lung. Which action is the nurse's most appropriate action? preparing the client for an emergency thoracotomy preparing the client for a pericardiocentesis preparing the client for a tracheostomy preparing the client for a chest tube insertion

preparing the client for a chest tube insertion The client's history and absent breath sounds suggest a right-sided pneumothorax or hemothorax, which will require treatment with a chest tube. The other options would not be appropriate actions.

The nurse is caring for a client following surgery. Which nursing action does not aid in meeting the postoperative goal of clear breath sounds? using an incentive spirometer providing a minimum of 1,000 mL of fluid per day assisting with early ambulation offering pain relief before having the client cough

providing a minimum of 1,000 mL of fluid per day After surgery, the client should drink a minimum of 2,500 mL of fluid per day (not 1,000 mL) to keep secretions liquefied and easier to cough up and eliminate from the upper respiratory tract. The client should use pain medication before coughing. The client should use the incentive spirometer every 2 to 4 hours. The nurse should monitor the client's breath sounds and temperature to detect early signs of infection. The nurse should assist with early ambulation.

A client presents to a physician's office complaining of dyspnea with exertion, weakness, and coughing up blood. Further examination reveals peripheral edema, crackles, and jugular vein distention. The nurse anticipates the physician will make which diagnosis? empyema pulmonary hypertension pulmonary tuberculosis chronic obstructive pulmonary disease (COPD)

pulmonary hypertension Dyspnea, weakness, hemoptysis, and right-sided heart failure are all signs of pulmonary hypertension. Clients with COPD present with chronic cough, dyspnea on exertion, and sputum production. Those with empyema are acutely ill and have signs of acute respiratory infection or pneumonia. Clients with pulmonary tuberculosis usually present with low-grade fever, night sweats, fatigue, cough, and weight loss.

The nurse is caring for a client who states an increase in dyspnea. Which intervention would the nurse perform first?

pulse oximeter Assessment is the first step in the nursing process. Assessing the pulse oximeter reading provides valuable information on the client's condition. Once the information is known, obtaining a breathing treatment or applying an oxygenated facemask, especially for a pulse oximeter reading under 90%, is appropriate. Health care provider notification would also be necessary as oxygen is a medication requiring an order.

To help control pain during coughing for a client who has had a pulmonary lobectomy, the nurse should: raise the bed to complete Fowler's position and help the client turn onto the operative side to splint the incision. keep the bed flat and tell the client to place the hands over the incision before taking a deep breath. raise the bed to semi-Fowler's position and position the client's hands so that the incision is supported anteriorly and posteriorly. place the bed in slight Trendelenburg's position and help the client turn onto the operative side to splint the incision.

raise the bed to semi-Fowler's position and position the client's hands so that the incision is supported anteriorly and posteriorly. Semi-Fowler's position allows for downward displacement of the diaphragm and relaxation of the abdominal muscles, which are needed for good ventilatory excursion. The hand placement supports the operative area and splints it without causing pain from pressure.Trendelenburg's position is contraindicated because abdominal contents pushing against the diaphragm will decrease effective lung volume.Keeping the bed flat does not allow the diaphragm to descend.Positioning the client on the operative side prevents maximum inflation of the lung.

An expected outcome of theophylline ethylenediamine when administered to a client with chronic obstructive pulmonary disease is: relax bronchial smooth muscle. reduce bronchial secretions. decrease alveolar elasticity. strengthen myocardial contractions.

relax bronchial smooth muscle. Theophylline ethylenediamine is a xanthine derivative that acts directly on bronchial smooth muscle to relax and dilate the bronchi and relieve bronchial constriction and spasms. When the drug exerts its primary desired effect, dyspnea and shortness of breath decrease.Theophylline ethylenediamine does not reduce bronchial secretions or decrease alveolar elasticity.Theophylline ethylenediamine does increase strength of myocardial contractility, but this is not the action for which it is used.

A client's pulmonary function tests note an increased residual volume and a decreased vital capacity. Which is the best nursing diagnosis? risk for fluid volume deficit impaired physical mobility risk for activity intolerance altered health maintenance

risk for activity intolerance These findings indicate respiratory disease; this client will have shortness of breath with exertion because of the trapped air. The client may have impaired physical mobility because of the inability to tolerate activities. Altered health maintenance or risk for fluid volume deficit are not supported by the test results.

A client had surgery for a deviated nasal septum. Which finding would indicate that bleeding is occurring even if the nasal drip pad remains dry and intact? increased pain nausea increased respiratory rate repeated swallowing

repeated swallowing Because of the dense nasal packing, bleeding may not be apparent through the nasal drip pad. Instead, the blood may run down the throat, causing the client to swallow frequently. The back of the throat, where the blood will be apparent, can be assessed with a flashlight. An accumulation of blood in the stomach can cause nausea and vomiting, but nausea would not be the initial indicator of bleeding. An increased respiratory rate occurs in shock but is not an early sign of bleeding in a client who has undergone nasal surgery. Increased pain warrants further assessment but is not an indicator of bleeding.

As status asthmaticus worsens, the nurse would expect the client to experience which acid-base imbalance? metabolic acidosis respiratory alkalosis metabolic alkalosis respiratory acidosis

respiratory acidosis As status asthmaticus worsens, the PaCO increases and the pH decreases, reflecting respiratory acidosis.

In chronic obstructive pulmonary disease (COPD), decreased carbon dioxide elimination results in increased carbon dioxide tension in arterial blood, leading to what acid-base imbalance? respiratory alkalosis metabolic alkalosis respiratory acidosis metabolic acidosis

respiratory acidosis Increased carbon dioxide tension in arterial blood leads to respiratory acidosis and chronic respiratory failure. In acute illness, worsening hypercapnia can lead to acute respiratory failure. The other acid-base imbalances would not correlate with COPD.

A nurse is reviewing arterial blood gas results on an assigned client. The pH is 7.32 with PCO2 of 49 mm Hg and a HCO3−of 28 mEq/L. The nurse reports to the physician which finding? metabolic alkalosis respiratory acidosis metabolic acidosis respiratory alkalosis

respiratory acidosis Respiratory acidosis would be reported to the physician citing the lab values. Analysis of the blood gases reveals that the client is acidotic with a pH under 7.35. Also noted is the PCO2above the normal range of 30 to 40 mm Hg. The HCO3− is slightly elevated because the normal level is 22 to 26 mEq/L.

A client has been hospitalized with myxedema coma. What acid-base imbalance would be expected in this client? metabolic acidosis respiratory stress respiratory alkalosis respiratory acidosis

respiratory acidosis The client's respiratory drive is depressed, resulting in alveolar hypoventilation, progressive carbon dioxide retention, narcosis, and coma. These symptoms, along with cardiovascular collapse and shock, require aggressive and intensive therapy if the client is to survive.

A client who is being treated for pneumonia reports sudden shortness of breath. An arterial blood gas (ABG) is drawn, yielding the following values: pH 7.21, PaCO2 64 mm Hg, HCO3 = 24 mm Hg. What does the ABG reflect? respiratory acidosis metabolic alkalosis respiratory alkalosis metabolic acidosis

respiratory acidosis The pH is below 7.40, PaCO2 is greater than 40, and the HCO3 is normal; therefore, respiratory acidosis exists and compensation by the kidneys has not begun, which indicates this was probably an acute event. The HCO3 of 24 is within the normal range so it is not metabolic alkalosis. The pH of 7.21 indicates acidosis, not alkalosis. The pH of 7.21 indicates it is an acidosis but the HCO3 of 24 is within the normal range, ruling out metabolic acidosis.

A client comes to the emergency department with status asthmaticus. The client's respiratory rate is 48 breaths/minute, and the client is wheezing. An arterial blood gas analysis reveals a pH of 7.52, a partial pressure of arterial carbon dioxide (PaCO2) of 30 mm Hg, PaO2 of 70 mm Hg, and bicarbonate (HCO3--) of 26 mEq/L. What disorder is indicated by these findings? metabolic acidosis metabolic alkalosis respiratory alkalosis respiratory acidosis

respiratory alkalosis Respiratory alkalosis results from alveolar hyperventilation. It's marked by a decrease in PaCO2 to less than 35 mm Hg and an increase in blood pH over 7.45. Metabolic acidosis is marked by a decrease in HCO3-- to less than 22 mEq/L, and a decrease in blood pH to less than 7.35. In respiratory acidosis, the pH is less than 7.35 and the PaCO2 is greater than 45 mm Hg. In metabolic alkalosis, the HCO3-- is greater than 26 mEq/L and the pH is greater than 7.45.

A client with a longstanding diagnosis of generalized anxiety disorder presents to the emergency room. The triage nurse notes upon assessment that the patient is hyperventilating. The triage nurse is aware that hyperventilation is the most common cause of: increased PaCO2. metabolic acidosis. acute CNS disturbances. respiratory alkalosis.

respiratory alkalosis. Extreme anxiety can lead to hyperventilation, which is the most common cause of acute respiratory alkalosis. Hyperventilation may be a response to metabolic acidosis, as the body attempts to rid itself of excess CO2; hyperventilation does not cause metabolic acidosis. Increased CO2 levels are associated with acidosis, not alkalosis. There are a number of potential causes of acute CNS disturbances.

A client admitted with acute anxiety has the following arterial blood gas (ABG) values: pH, 7.55; partial pressure of arterial oxygen (PaO2), 90 mm Hg; partial pressure of arterial carbon dioxide (PaCO2), 27 mm Hg; and bicarbonate (HCO3-), 24 mEq/L. Based on these values, the nurse suspects respiratory alkalosis. metabolic acidosis. respiratory acidosis. metabolic alkalosis.

respiratory alkalosis. This client's above-normal pH value indicates alkalosis. The below-normal PaCO2 value indicates acid loss via hyperventilation; this type of acid loss occurs only in respiratory alkalosis. These ABG values wouldn't occur in metabolic acidosis, respiratory acidosis, or metabolic alkalosis.

Which outcome criteria would the nurse develop for a child with cystic fibrosis who has ineffective airway clearance related to increased pulmonary secretions and inability to expectorate? ability to tolerate usual diet without vomiting absence of chills and fever respiratory rate and rhythm within expected range ability to engage in age-related activities

respiratory rate and rhythm within expected range After treatment, the client outcome would be that respiratory status would be within normal limits, as evidenced by a respiratory rate and rhythm within expected range. Absence of chills and fever, although related to an underlying problem causing the respiratory problem (e.g., the infection), do not specifically relate to the respiratory problem of ineffective airway clearance. The child's ability to engage in age-related activities may provide some evidence of improved respiratory status. However, this outcome criterion is more directly related to activity intolerance. Although the child's ability to tolerate his or her usual diet may indirectly relate to respiratory function, this outcome is more specifically related to an imbalanced nutrition that may or may not be related to the child's respiratory status.

The health care provider (HCP) has prescribed pseudoephedrine. The nurse should instruct the client about which possible adverse effect of this drug? diplopia constipation restlessness bradycardia

restlessness Adverse effects of pseudoephedrine are experienced primarily in the cardiovascular system and through sympathetic effects on the central nervous system (CNS). The most common CNS adverse effects include restlessness, dizziness, tension, anxiety, insomnia, and weakness. Common cardiovascular adverse effects include tachycardia, hypertension, palpitations, and arrhythmias. Constipation and diplopia are not adverse effects of pseudoephedrine. Tachycardia, not bradycardia, is an adverse effect of pseudoephedrine.

The nurse is assessing a client recovering from anesthesia. Which finding is an early indicator of hypoxemia? urgency restlessness somnolence chills

restlessness One of the earliest signs of hypoxia is restlessness and agitation. Decreased level of consciousness and somnolence are later signs of hypoxia. Chills can be related to the anesthetic agent used but are not indicative of hypoxia. Urgency is not related to hypoxia.

Which finding alerts the nurse to possible internal bleeding in a client who has undergone pulmonary lobectomy 2 days ago? increased blood pressure and decreased pulse and respiratory rates restlessness and shortness of breath sanguineous drainage from the chest tube at a rate of 50 ml/hour during the past 3 hours urine output of 180 mL during the past 3 hours

restlessness and shortness of breath Restlessness indicates cerebral hypoxia due to decreased circulating volume. Shortness of breath occurs because blood collecting in the pleural space faster than suction can remove it prevents the lung from reexpanding.Increased blood pressure and decreased pulse and respiratory rates are classic late signs of increased intracranial pressure. Decreasing blood pressure and increasing pulse and respiratory rates occur with hypovolemic shock.Sanguineous drainage that changes to serosanguineous drainage at a rate less than 100 mL/h is normal in the early postoperative period.Urine output of 180 mL over the past 3 hours indicates normal kidney perfusion.

A client with a diagnosis of respiratory acidosis is experiencing renal compensation. What function does the kidney perform to assist in restoring acid-base balance? returning bicarbonate to the body's circulation returning acid to the body's circulation sequestering free hydrogen ions in the nephrons excreting bicarbonate in the urine

returning bicarbonate to the body's circulation The kidney performs two major functions to assist in acid-base balance. The first is to reabsorb and return to the body's circulation any bicarbonate from the urinary filtrate; the second is to excrete acid in the urine. Retaining bicarbonate will counteract an acidotic state. The nephrons do not sequester free hydrogen ions.

A nurse is weaning a client from mechanical ventilation. Which assessment finding indicates the weaning process should be stopped? runs of ventricular tachycardia oxygen saturation of 93% blood pressure increase from 120/74 mm Hg to 134/80 mm Hg respiratory rate of 16 breaths/minute

runs of ventricular tachycardia Ventricular tachycardia indicates that the client isn't tolerating the weaning process. The weaning process should be stopped before lethal ventricular arrhythmias occur. A respiratory rate of 16 breaths/minute and an oxygen saturation of 93% are normal findings. Although the client's blood pressure has increased, it hasn't increased more than 20% over baseline, which would indicate that the client isn't tolerating the weaning process.

A client asks a nurse a question about the tuberculin skin test for tuberculosis. The nurse should base their response on the fact that the skin test doesn't differentiate between active and dormant tuberculosis infection. presence of a wheal at the injection site in 2 days indicates active tuberculosis. area of redness is measured in 3 days and determines whether tuberculosis is present. test stimulates a reddened response in some clients and requires a second test in 3 months.

skin test doesn't differentiate between active and dormant tuberculosis infection. The tuberculin skin test doesn't differentiate between active and dormant infections. If a positive reaction occurs, a sputum smear and culture as well as a chest X-ray are necessary to provide more information. Although the area of redness is measured in 3 days, a second test may be needed; neither test indicates that tuberculosis is active. In the tuberculin skin test, an induration 5 to 9 mm in diameter indicates a borderline reaction; a larger induration indicates a positive reaction. The presence of a wheal within 2 days doesn't indicate active tuberculosis.

A client with bacterial pneumonia is to be started on IV antibiotics. The nurse should verify that which diagnostic test has been completed before administering the antibiotic? red blood cell count sputum culture chest radiograph urinalysis

sputum culture A sputum specimen is obtained for culture to determine the causative organism. After the organism is identified, an appropriate antibiotic can be prescribed. Beginning antibiotic therapy before obtaining the sputum specimen may alter the results of the test. Urinalysis, a chest radiograph, and a red blood cell count do not need to be obtained before initiation of antibiotic therapy for pneumonia.

The nurse is admitting a client who just had a bronchoscopy. Which assessment should be the nurse's priority? ability to deep breathe presence of carotid pulse medication allergies swallow reflex

swallow reflex The physician sprays a local anesthetic into the client's throat before performing a bronchoscopy. The nurse must assess the swallow reflex when the client returns to the unit and before giving them anything by mouth. The nurse should also assess for medication allergies, carotid pulse, and deep breathing, but they aren't the priority at this time.

The client with tuberculosis is to be discharged home with nursing follow-up. Which aspect of nursing care will have the highest priority? assessing the client's environment for sanitation teaching the client about the disease and its treatment offering the client emotional support coordinating various agency services

teaching the client about the disease and its treatment Ensuring that the client is well educated about tuberculosis is the highest priority. Education of the client and family is essential to help the client understand the need for completing the prescribed drug therapy to cure the disease. Offering the client emotional support, coordinating various agency services, and assessing the environment may be part of the care for the client with tuberculosis; however, these interventions are of less importance than education about the disease process and its treatment.

For a client with chronic obstructive pulmonary disease, which nursing intervention helps maintain a patent airway? administering ordered sedatives regularly limiting oral fluid intake teaching the client how to perform controlled coughing assisting with feeding

teaching the client how to perform controlled coughing Controlled coughing helps maintain a patent airway by helping to mobilize and remove secretions. A moderate fluid intake (usually 2 L or more daily) and moderate activity help liquefy and mobilize secretions. Bed rest and sedatives may limit the client's ability to maintain a patent airway, causing a high risk of infection from pooled secretions.

Which is an expected outcome for a client who has been treated for bacterial pneumonia? a respiratory rate of 25 to 30 breaths/min chest pain that is minimized by splinting the rib cage a maximum loss of 5 to 10 lb (2 to 5 kg) of body weight the ability to perform activities of daily living without dyspnea

the ability to perform activities of daily living without dyspnea An expected outcome for a client recovering from pneumonia would be the ability to perform activities of daily living without experiencing dyspnea. A respiratory rate of 25 to 30 breaths/min indicates the client is experiencing tachypnea, which would not be expected on recovery. A weight loss of 5 to 10 lb (2 to 5 kg) is undesirable; the expected outcome would be to maintain normal weight. A client who is recovering from pneumonia should experience decreased or no chest pain.

A client's chest tube is connected to a drainage system with a water seal. The nurse notes that the fluid in the water seal column is fluctuating with each breath that the client takes. How should the nurse interpret this finding? Fluctuation means that: the client is developing subcutaneous emphysema. there is an obstruction in the chest tube. there is a leak in the chest tube system. the chest tube system is functioning properly.

the chest tube system is functioning properly. Fluctuation of fluid with respirations in the water seal column indicates that the system is functioning properly. If an obstruction were present in the chest tube, fluid fluctuation would be absent. Subcutaneous emphysema occurs when air pockets can be palpated beneath the client's skin around the chest tube insertion site. A leak in the system is indicated when bubbling occurs in the water seal column.

The nurse is prioritizing care for several clients. Which client should the nurse assess first? the client with bilateral wheezing receiving a breathing treatment the client with stridor who just received the first dose of an antibiotic the client with chest pain improving after medication the client with a blood pressure of 150/90 mmHg

the client with stridor who just received the first dose of an antibiotic The highest priority client is the client with stridor who started an antibiotic. Stridor is an assessment finding indicating an extremely narrowed airway. This may indicate an anaphylactic reaction to the antibiotic. The nurse must intervene to prevent anaphylactic shock. The airway is the top priority. Next, the nurse should assess the client with wheezing. Finally, the clients with improving chest pain and elevated blood pressure should be assessed.

A nurse on the medical-surgical unit just received the client care assignment report. Which client should the nurse assess first? the client with crackles and fever who reports pleuritic pain the client who had difficulty sleeping, daytime fatigue, and morning headache the client with unilateral leg swelling who reports anxiety and shortness of breath the client with anorexia, weight loss, and night sweats

the client with unilateral leg swelling who reports anxiety and shortness of breath The client who reports anxiety and shortness of breath and has unilateral leg swelling should be seen first. This client is exhibiting signs and symptoms of pulmonary embolism, which is a life-threatening condition. Crackles, fever, and pleuritic pain are signs and symptoms of pneumonia. Anorexia, weight loss, and night sweats are signs and symptoms of tuberculosis. Difficulty sleeping, daytime fatigue, and morning headache are symptoms of sleep apnea. Pneumonia, sleep apnea, and tuberculosis aren't medical emergencies. Clients with these disorders don't take priority over the client with a pulmonary embolism.

A client with chronic obstructive pulmonary disease tells a nurse that they feel short of breath. The client's respiratory rate is 36 breaths/minute and the nurse auscultates diffuse wheezes. The client's arterial oxygen saturation is 84%. The nurse calls the assigned respiratory therapist to administer an ordered nebulizer treatment. The therapist says, "I have several more nebulizer treatments to do on the unit where I am now. As soon as I'm finished, I'll come and assess the client." The nurse's most appropriate action is to the nurse gives the nebulizer treatment. administer the treatment by metered-dose inhaler. stay with the client until the therapist arrives. notify the primary physician immediately.

the nurse gives the nebulizer treatment. The client's needs are preeminent, so the nurse should administer the nebulizer treatment immediately. The nurse can deal with the respiratory therapist's lack of response after the client's condition is stabilized. There is no need to involve the physician in personnel issues. Staying with the client is important, but it isn't a substitute for administering the needed bronchodilator. The order is for a nebulizer treatment not a metered-dose inhaler, so the nurse can't change the route without a new order from the physician.

A client who had a left thoracoscopy sustained an injury secondary to the surgery position. The nurse should assess the client for which sign? foot drop knee swelling and pain tingling in the arm absence of the Achilles reflex

tingling in the arm A client who had a left thoracoscopy is placed in the lateral position, in which the most common injury is an injury to the brachial plexus. Numbness and tingling in the arm suggests a brachial plexus injury. There is no undue pressure on the ankles or knees during thoracic surgery.

A client is receiving furosemide as part of the treatment for heart failure. Which assessment finding indicates that the medication is attaining a therapeutic effect? crackles auscultated halfway up lungs, previously in bases trace peripheral edema, previously +2 blood pressure 140/80 mm Hg PaO2 80 mm Hg

trace peripheral edema, previously +2 The therapeutic effect of furosemide is to mobilize excess fluid. The client's peripheral edema should decrease, indicated by changing from +2 to trace. As furosemide decreases fluid in the lungs, the client's crackles should decrease, not continue to progress. If furosemide is attaining a therapeutic effect, the blood pressure should decrease into normal range and the oxygen level should increase to above 90%.

A client recovering from a pulmonary embolism is receiving warfarin. To counteract a warfarin overdose, the nurse should administer vitamin K1 (phytonadione). protamine sulfate. vitamin C. heparin.

vitamin K1 (phytonadione). Vitamin K1 is the antidote for a warfarin overdose. Heparin is a parenteral anticoagulant. Vitamin C isn't an antidote. Protamine sulfate is the antidote for heparin.

A nurse is caring for a client admitted with an exacerbation of asthma. The nurse knows the client's condition is worsening when the client: asks for an additional pillow. wants the head of the bed raised to a 90-degree level. has a pulse oximetry reading of 91%. uses the sternocleidomastoid muscles.

uses the sternocleidomastoid muscles. Use of accessory muscles indicates worsening breathing conditions. Asking for an additional pillow, having a 91% pulse oximetry reading, and requesting the nurse to raise the head of the bed are not indications of a worsening condition.

For a client with advanced chronic obstructive pulmonary disease (COPD), which nursing action best promotes adequate gas exchange? keeping the client in low-Fowler's position using a Venturi mask to deliver oxygen as ordered administering a sedative as ordered encouraging the client to drink 3 glasses of fluid daily

using a Venturi mask to deliver oxygen as ordered A Venturi mask allows the nurse to deliver a specified, controlled amount of oxygen consistently and accurately. Drinking 3 glasses of fluid daily wouldn't affect gas exchange or be sufficient to liquefy secretions, which are common in COPD. Clients with COPD and respiratory distress should be placed in high Fowler's position and shouldn't receive sedatives or other drugs that may further depress the respiratory center.

A client reports having a dry, hacking cough that disturbs sleep at night. Which antitussive agent and intervention are most appropriate for this client? using a cooling mist humidifier and administering dextromethorphan decreasing the room temperature and administering a benzonatate increasing fluids to liquefy secretions and administering codeine providing a heat vaporizer and administering hydrocodone

using a cooling mist humidifier and administering dextromethorphan Dextromethorphan is the most widely used antitussive in Canada because it produces few adverse reactions while effectively suppressing a cough. A cool mist humidifier will help open nasal passages. Benzonatate is used for cough associated with respiratory conditions and chronic pulmonary diseases. Opioid antitussives, such as codeine and hydrocodone, are reserved for treating unruly coughs usually associated with lung cancer.

A nurse is caring for a client who is at high risk for developing pneumonia. Which intervention should the nurse include on the client's care plan? using strict hand hygiene keeping the head of the bed at 15 degrees or less providing oral hygiene daily turning the client every 4 hours to prevent fatigue

using strict hand hygiene The nurse should use strict hand hygiene to help minimize the client's exposure to infection, which could lead to pneumonia. The head of the bed should be kept at a minimum of 30 degrees. The client should be turned and repositioned at least every 2 hours to help promote secretion drainage. Oral hygiene should be performed every 4 hours to help decrease the number of organisms in the client's mouth that could lead to pneumonia.

A healthy client presents to the clinic for a routine examination. When auscultating the client's lower lung lobes, the nurse should expect to hear which type of breath sound? tracheal bronchovesicular vesicular bronchial

vesicular Vesicular breath sounds are soft, low-pitched sounds normally heard over the lower lobes of the lung. They're prolonged on inhalation and shortened on exhalation. Bronchial breath sounds are loud, high-pitched sounds normally heard next to the trachea; discontinuous, they're loudest during expiration. Tracheal breath sounds are harsh, discontinuous sounds heard over the trachea during inhalation or exhalation. Bronchovesicular breath sounds are medium-pitched, continuous sounds that occur during inhalation or exhalation. They're best heard over the upper third of the sternum and between the scapulae.

A nurse is caring for a client with tuberculosis. Which infection-control technique is the priority when caring for this client? performing hand hygiene before entering the client's room wearing an isolation gown, mask, and gloves when caring for the client entering the client's room only every 2 hours wearing an N95 respirator when caring for the client

wearing an N95 respirator when caring for the client Because tuberculosis is transmitted via airborne droplets, the priority for nurses caring for this client is to wear an N95 respirator whenever entering the client's room. Performing hand hygiene before entering the room will not prevent the transmission of TB. When using a fit-tested N95 respirator, it is not necessary to limit the time spent with the client. Isolation gowns are not necessary for airborne isolation.

The nurse is conducting a health history with a client with active tuberculosis. The nurse should ask the client about: increased appetite. mental status changes. dyspnea on exertion. weight loss.

weight loss. Tuberculosis typically produces anorexia and weight loss. Other signs and symptoms may include fatigue, low-grade fever, and night sweats.Increased appetite is not a symptom of tuberculosis; dyspnea on exertion and change in mental status are not common symptoms of tuberculosis.


Related study sets

HESI NU 112 REVIEW- NURSING CONCEPTS (BASIC CARE AND COMFORT)

View Set

Hist 101 - Learning Curves Chp 7

View Set

Chapter 52: Assessment and Management of Patient with Endocrine Disorders Prep U

View Set

11.3 Physical Science Reg. Notes

View Set